2019 jpip-a 病理診断生涯教育プログラムleslie dodd, md fcap sabrina c. sopha, md fcap...

98
2019 JPIP-A 病理診断生涯教育プログラム Performance Improvement Program in Surgical Pathology

Upload: others

Post on 26-Dec-2019

1 views

Category:

Documents


0 download

TRANSCRIPT

Page 1: 2019 JPIP-A 病理診断生涯教育プログラムLeslie Dodd, MD FCAP Sabrina C. Sopha, MD FCAP Raul Gonzalez, MD FCAP Nichole L. Steidler, MD FCAP Christina Isacson, MD FCAP Justin

2019 JPIP-A 病理診断生涯教育プログラム Performance Improvement Program in Surgical Pathology

Page 2: 2019 JPIP-A 病理診断生涯教育プログラムLeslie Dodd, MD FCAP Sabrina C. Sopha, MD FCAP Raul Gonzalez, MD FCAP Nichole L. Steidler, MD FCAP Christina Isacson, MD FCAP Justin

PaLaNA Initiative

SURGICAL PATHOLOGY COMMITTEE

Rochelle A. Simon, MD FCAP, Chair

Aaron Auerbach, MD FCAP, Vice Chair

Oyedele A. Adeyi, MBBS FCAP

Vijayalakshmi Ananthanarayanan, MD FCAP

Nilsa C. Ramirez, MD FCAP

Kirtee Raparia, MD FCAP

Phyu P Aung, MD FCAP Vijaya B. Reddy MD, MBA FCAP

Deborah Ann Belchis, MD FCAP

Rajan Dewar, MD, PhD FCAP

Safia N. Salaria, MD FCAP

Robert Alan Schwartz, MD FCAP

Leslie Dodd, MD FCAP Sabrina C. Sopha, MD FCAP

Raul Gonzalez, MD FCAP Nichole L. Steidler, MD FCAP

Christina Isacson, MD FCAP Justin M. Wells, MD FCA

Xiaoyin Jiang, MD FCAP

Archi Patel, MD

Danielle E. Westfall, MD FCAP

Page 3: 2019 JPIP-A 病理診断生涯教育プログラムLeslie Dodd, MD FCAP Sabrina C. Sopha, MD FCAP Raul Gonzalez, MD FCAP Nichole L. Steidler, MD FCAP Christina Isacson, MD FCAP Justin

(Table of Contents)

2019-01 5

(Case 2019-01) Nephroblastoma with heterologous elements teratoid Wilms’ tumor (4)

2019-02 19

(Case 2019-02) Fibroma (18)

2019-03 27

(Case 2019-03) Mitochondrial cardiomyopathy (26)

2019-04 TFE3 35

(Case 2019-04) TFE3-associated renal cell carcinoma (34)

2019-05 45

(Case 2019-05) Hepatocellular carcinoma (44)

2019-06 55

(Case 2019-06) Poorly differentiated squamous cell carcinoma (54)

2019-07 61

(Case 2019-07) Seminoma (60)

2019-08 71

(Case 2019-08) Well-differentiated liposarcoma (70)

2019-09 79

(Case 2019-09) Embryonal rhabdomyosarcoma (78)

2019-10 89

(Case 2019-10) Anaplastic thyroid carcinoma (88)

97

Educational Questions Answer Key (96)

3

Page 4: 2019 JPIP-A 病理診断生涯教育プログラムLeslie Dodd, MD FCAP Sabrina C. Sopha, MD FCAP Raul Gonzalez, MD FCAP Nichole L. Steidler, MD FCAP Christina Isacson, MD FCAP Justin

Case # 2019-01

Diagnosis Nephroblastoma with heterologous elements (teratoid Wilms’ tumor)

Site Left kidney

Clinical

Summary

An otherwise healthy 2-year-old girl is taken to the pediatrician after her mother notes a relatively sudden increase in abdominal girth. Upon physical examination, the pediatrician palpates a large abdominal mass and orders a series of tests, including a whole abdomen computed tomography (CT) scan. The CT scan confirms the presence of a heterogeneous cystic left renal mass with enhancing soft tissue and adipose tissue components, and excludes evidence of lymphadenopathy and/or metastatic deposits. The patient undergoes surgery. The well-circumscribed intrarenal mass involves the upper pole of the left kidney, measures 9.0 x 7.0 x 6.0 cm, and exhibits a variegated pink-tan, soft, and friable cut surface; there are areas of hemorrhage involving 20% of the cut surface, as well as patchy yellow-tan and soft areas.

Master List 2388 Cellular mesoblastic nephroma1020 Cystic partially-differentiated nephroblastoma5596 Intrarenal teratoma1666 Metanephric adenoma5597 Nephroblastoma with heterologous elements (teratoid Wilms’ tumor)1012 Neuroblastoma

Criteria for

Diagnosis

and

Comments

Upon histologic evaluation, the renal tumor is characterized by a triphasic proliferation of blastemal, stromal, and epithelial neoplastic elements associated with a heterologous component that includes mature adipose tissue, glandular epithelium, and skeletal muscle. The blastemal component is slightly dominant and characterized by a proliferation of ‘‘small blue cells’’ mainly arranged in a nodular pattern, with high nuclear-to-cytoplasmic ratio, coarse chromatin, and brisk mitotic activity. The minor epithelial component consists mainly of tubules lined by cuboidal to columnar primitive cells. The stromal component is mostly myxoid and variably edematous with fibrocytic differentiation. Mature heterologous elements are interspersed with the triphasic neoplastic elements and include adipose tissue, glandular tissue, and a very minor component of skeletal muscle. Although the epithelial lining of the majority of the simple glands consists of columnar ciliated epithelium and transitional epithelium, in rare areas, intestinal- type epithelium (including goblet cells) is noted. Microscopic calcifications (mainly within the epithelial component), areas of hemorrhage, andareas of tumor necrosis are noted.

In the United States, nephroblastoma (Wilms’ tumor or WT) is the most common primary

4

Page 5: 2019 JPIP-A 病理診断生涯教育プログラムLeslie Dodd, MD FCAP Sabrina C. Sopha, MD FCAP Raul Gonzalez, MD FCAP Nichole L. Steidler, MD FCAP Christina Isacson, MD FCAP Justin

# 2019-01

Nephroblastoma with heterologous elements (teratoid Wilms’ tumor)

2CT

CT

9.0 x 7.0 x 6.0 cm20

(2388) Cellular mesoblastic nephroma(1020) Cystic partially-differentiated nephroblastoma(5596) Intrarenal teratoma(1666) Metanephric adenoma(5597)

Nephroblastoma with heterologous elements (teratoid Wilms’ tumor)(1012) Neuroblastoma

3

small blue cells N/C

nephroblastoma Wilms’ tumor WT

5

Page 6: 2019 JPIP-A 病理診断生涯教育プログラムLeslie Dodd, MD FCAP Sabrina C. Sopha, MD FCAP Raul Gonzalez, MD FCAP Nichole L. Steidler, MD FCAP Christina Isacson, MD FCAP Justin

malignant renal tumor in the pediatric age group. The peak incidence of this neoplasm is between 2-5 years of age, with approximately 95% of cases occurring before the age of 10. WT is slightly more common in girls and is rarely reported during the neonatal period and in adults. Rare extrarenal WT (eg, arising in various locations including the female reproductive tract, the thorax, and the paratesticular region) are also reported in the literature. The large majority of renal WTs are unifocal, although multifocal tumors can be seen in up to 7% of cases and bilateral tumors in up to 5%. In close to 10% of cases, WTs arise in association with 3 syndromes: WAGR (Wilms tumor, aniridia, genital anomalies, and mental retardation) syndrome, Denys---Drash syndrome (gonadal dysgenesis and early onset nephropathy), and Beckwith---Wiedemann syndrome (organomegaly, macroglossia, hemihypertrophy, omphalocele, and adrenal cytomegaly).

Although WT usually presents as a painless abdominal mass, in close to a third of cases the clinical presentation may include anorexia, pain, and vomiting. If present, paraneoplastic signs and symptoms can include hypertension, erythrocytosis, hypercalcemia, and Cushing syndrome. In some patients (especially children), renal masses can be detected by palpation. Further preoperative characterization of a retroperitoneal mass can be achieved with the assistance of a variety of radiological studies depending on the patient’s age.

Grossly, WT usually presents as a single well-circumscribed mass with a pseudocapsule that measures more than 5.0 cm in diameter; the median weight is 550 grams, and it can arise in any area of the kidney. The cut surface is lobulated, pale gray or pink, soft, and friable. Common gross findings include hemorrhage, necrosis, calcification, and cystic change. Extension into the renal pelvis and ureter can be seen, and invasion of the renal vein is often identified.

Microscopically, WT with classical triphasic components (blastemal, stromal, and epithelial cell types) are the most common (41%); however, when a component predominates and accounts for more than two-thirds of the tumor, then it is classified accordingly. The blastema-predominant tumor variant is the most common (close to 39%), and the stroma-predominant one is the rarest (1.4%). Biphasic and monophasic tumor variants can also be seen. When diffuse differentiation toward skeletal muscle comprises >50% of the tumor volume, the term fetal rhabdomyomatous nephroblastoma (a tumor with relative resistance to chemotherapy) is used.

WT with heterologous elements, also known as teratoid Wilms tumor (TWT), is a rare histological variant. Although the term TWT is not well-defined or widely used, it describes a WT with heterologous epithelial and stromal differentiation that comprises >50% of the tumor volume. Current literature on this subject is limited to small case series and case reports with variable data elements that restrict comparisons with the larger ‘‘classic WT’’ series. Some investigators state that TWTs respond well to surgical therapy, present as bilateral tumors in close to one-third of cases, demonstrate poor radiologic response to preoperative chemotherapy, and rarely present at extra-renal sites. Reports that TWTs (when compared to WTs) have similar sex and age distribution, similar overall prognosis, and are of a lower grade

6

Page 7: 2019 JPIP-A 病理診断生涯教育プログラムLeslie Dodd, MD FCAP Sabrina C. Sopha, MD FCAP Raul Gonzalez, MD FCAP Nichole L. Steidler, MD FCAP Christina Isacson, MD FCAP Justin

2 5 95 10WT

WTWT 7 5

10 WT 3 WAGRDenys–Drash

Beckwith–Wiedemann

WT 3 1

WT 5.0 cm550g

WT41 3 2

391.4

50 rhabdomyomatous nephroblastoma

teratoid Wilms tumor TWT WTTWT

50 WTWT

TWT 3 1

TWT WT

CT TWT

7

Page 8: 2019 JPIP-A 病理診断生涯教育プログラムLeslie Dodd, MD FCAP Sabrina C. Sopha, MD FCAP Raul Gonzalez, MD FCAP Nichole L. Steidler, MD FCAP Christina Isacson, MD FCAP Justin

at the time of presentation also exist in the literature. In the proper clinical context, radiologic studies that suggest the presence of heterologous differentiation in a renal tumor (eg, adipose tissue presenting as reduced attenuation in a CT scan) may represent a TWT.

Extensive sampling is required (at least one tissue section per cm of the largest dimension of the tumor) in order to characterize WT. The undifferentiated mesonephric blastemal cells are ‘‘small blue cells’’ with high nuclear-to-cytoplasmic ratio, coarse chromatin, and brisk mitotic activity that may be arranged in nodular, serpentine, and basaloid patterns variably associated with a myxoid mesenchymal background; these patterns exhibit demarcated edges. The diffuse blastemal pattern has infiltrative margins. In the common nephrogenic epithelial cell type, primitive columnar to cuboidal cells are arranged in glomeruloid and/or tubular structures. The stromal cell types can be fibrocytic or myxoid or exhibit skeletal muscle differentiation. Heterologous epithelial differentiation (eg, mucinous, squamous,neuroepithelial, neuroendocrine), nephrogenic stromal differentiation (eg, myxoid, fibrous, smooth muscle, adipose tissue), and heterologous stromal differentiation (eg, skeletal muscle, cartilage, bone) can be also be seen. In TWTs, the most common heterologous element reported is adipose tissue; odontogenic epithelium and elements of salivary tissue, intestinal tract, and respiratory tract have also been identified.

The presence of anaplasia (defined as a threefold increase in nuclear diameter,hyperchromasia of the enlarged nuclei, and multipolar mitotic figures) is identified in up to 11% of WTs and indicates ‘‘unfavorable histology’’ that correlates with resistance to chemotherapy. In the World Health Organization Classification of Tumours of the Urinary System and Male Genital Organs, ‘‘focal anaplasia’’ refers to clearly defined areas of anaplastic features within the tumor, anaplasia confined to the renal parenchyma, the absence of anaplasia within vascular spaces, and no severe pleomorphism within non-anaplastic areas of tumor. Criteria for ‘‘diffuse anaplasia’’ in resected specimens include the presence of any of the following: multifocal anaplasia; extension of anaplasia through the tumor capsule; anaplastic tumor cells within intrarenal vessels, renal sinus, and metastatic or extracapsular sites; or anaplasia that is not clearly defined from the surrounding non-anaplastic tumor. Anaplasia is reported to be uncommon in TWT.

Immunohistochemical stains for several markers may assist in establishing the diagnosis of WT depending on the tumor components. Positive nuclear immunostaining for WT1 is noted in close to 90% of blastemal and primitive epithelial elements, but it may be negative in other tumor components. Positive immunostaining with cytokeratin stains, PAX2, and CD56 may also be demonstrated in some WT components. Use of the current cancer case summary/synoptic report for pediatric renal tumors developed by the College of American Pathologists is recommended when reporting WT.

The genetic alterations associated with WTs are numerous and complex and includeabnormalities of the WT1 -catenin gene CTNNB1 on chromosome 3p22, the WTX gene located at Xq11.1, and the TP53 tumor suppressor gene on chromosome 17p13.1; other reported alterations include loss of imprinting on 11p15 and

8

Page 9: 2019 JPIP-A 病理診断生涯教育プログラムLeslie Dodd, MD FCAP Sabrina C. Sopha, MD FCAP Raul Gonzalez, MD FCAP Nichole L. Steidler, MD FCAP Christina Isacson, MD FCAP Justin

WT 1 cmN/C

small blue cells

TWT

311 WT WHO

WHO Classification of Tumours of the Urinary System and Male Genital Organs

TWT

WTWT1 90

PAX2 CD56WT WT College of American

Pathologists summary/synoptic report

WT 11p13 WT1 3p22- CTNNB1 Xp11.1 WTX 17p13.1 TP53

11p151p 16q

9

Page 10: 2019 JPIP-A 病理診断生涯教育プログラムLeslie Dodd, MD FCAP Sabrina C. Sopha, MD FCAP Raul Gonzalez, MD FCAP Nichole L. Steidler, MD FCAP Christina Isacson, MD FCAP Justin

loss of heterozygosity of both chromosomes 1p and 16q. These genetic alterations affect many aspects of tumor formation, disease progression, and response to therapy, among others. Testing for their presence or absence can assist in the development of a comprehensive treatment plan and the determination of significant prognostic factors. Genetic alterations reported in association with TWT are similar to those associated with cases of classic WT.

The differential diagnosis of intrarenal WT in pediatric-age patients includes nephrogenic rests, cystic partially differentiated nephroblastoma, mesoblastic nephroma, metanephric adenoma, and several ‘‘small blue round cell tumors of childhood’’ (eg, neuroblastoma, extraosseous Ewing sarcoma) that can arise as primary intrarenal neoplasms.

Nephrogenic rests are small collections of abnormally persistent embryonal-type cells that can potentially give rise to WTs, become hyperplastic, or undergo fibrotic changes. Although not always associated with malignancy, they are encountered in 25%-40% of patients with WTs (though they appear to be less frequently noted in TWTs). Patients with nephrogenic rests are at an increased risk of developing WT in the contralateral kidney. When multifocal or diffuse, they are referred to as nephroblastomatosis. Nephrogenic rests exhibit variable amounts of epithelial, blastemal, and stromal components and are classified as either perilobar or intralobar. Perilobar rests are typically blastemal-rich, well-demarcated, and situated at the periphery of the renal lobe. Intralobar rests tend to be epithelial- and stromalcell rich, are more infiltrative, and are situated more centrally in the lobe. Uponmicroscopic evaluation, it may be impossible to distinguish nephrogenic rests from WT, especially at high magnification. Therefore, it is important to correlate the histological findings with the clinical and radiological information when dealing with small specimens, such as biopsies.

Cystic partially differentiated nephroblastoma is a tumor of low malignant potential,considered to be a low-risk variant of WT. This rare, multilocular, and wellcircumscribed cystic tumor usually occurs in patients under 2 years of age; it is more common in boys, and it typically presents as a single unilateral renal tumor. Microscopic evaluation of the delicate tissue septa reveals spindle-shaped stromal cells variably associated with several elements, including blastema, nephroblastomatous epithelium, mesenchyme (differentiated and/or undifferentiated), skeletal muscle, cartilage, and adipose tissue. The cystic spaces can be devoid of an epithelial lining; however, flattened, cuboidal, or hobnail cells may line the cysts.

Mesoblastic nephroma (also known as congenital mesoblastic nephroma) is a tumor of low malignant potential that comprises 3%-5% of all childhood renal tumors. In cellular mesoblastic nephroma (66% of cases), the cells are spindle-shaped or polygonal, with high nuclear-to-cytoplasmic ratios; moderate nuclear pleomorphism may be present, and the cells may be arranged in a herringbone pattern. This highly cellular pattern exhibits brisk mitotic activity (8-30 mitoses/10 high-power fields) and a pushing border. At the ultrastructural level, it is identical to infantile fibrosarcoma (it has the same t(12;15)p13;q25 chromosomal translocation, resulting in ETV6-NTRK3 fusion gene rearrangement). This pattern has a more

10

Page 11: 2019 JPIP-A 病理診断生涯教育プログラムLeslie Dodd, MD FCAP Sabrina C. Sopha, MD FCAP Raul Gonzalez, MD FCAP Nichole L. Steidler, MD FCAP Christina Isacson, MD FCAP Justin

TWTWT

WT cystic partially-differentiated nephroblastoma mesoblastic nephromametanephric adenoma small blue round cell tumors neuroblastoma extraosseous Ewing sarcoma

WTWT

25 40 TWTWT

nephroblastomatosis perilobar intralobar

WT

Cystic partially-differentiated nephroblastomaWT 2

hobnail

mesoblastic nephroma3 5 cellular mesoblastic

nephroma 66 N/C

8 30 / 10 t(12;15) p13;q25

ETV6-NTRK34

11

Page 12: 2019 JPIP-A 病理診断生涯教育プログラムLeslie Dodd, MD FCAP Sabrina C. Sopha, MD FCAP Raul Gonzalez, MD FCAP Nichole L. Steidler, MD FCAP Christina Isacson, MD FCAP Justin

aggressive behavior than the classic pattern. The mean age at presentation is 4 months.

Metanephric adenoma is a rare benign renal epithelial tumor; it is more common in adults than in children and affects twice as many females as males. These tumors are unilateral and only rarely multifocal. Grossly, metanephric adenomas are well circumscribed and variably sized. They exhibit a tan to gray cut surface and may be soft or firm. These cellular tumors are comprised of small epithelial cells with a high nuclear-to-cytoplasmic ratio. The uniform basophilic nuclei vary from round to oval; the chromatin is delicate, and nucleoli are either absent or inconspicuous. The neoplastic cells can be arranged in small acini, tubules, and papillary structures; a solid cellular arrangement can also be seen. The scanty cytoplasm is eosinophilic. Significant mitotic activity is absent. The eosinophilic tumor stroma can be veryscanty, loose and edematous, or sclerotic. Osseous metaplasia can be identified in up to 20% of tumors; psammoma bodies are common, and dystrophic calcification can also be identified. More than 90% of metanephric adenomas have the BRAF V600E mutation. They are positive for CD57, unlike WT.

Intrarenal neuroblastomas are extremely rare tumors, so invasion of the kidney from an adjacent primary adrenal gland tumor needs to be excluded. Neuroblastomas are composed of small round to oval neoplastic cells with scanty cytoplasm, round to oval nuclei with ‘‘salt-and-pepper’’ chromatin, and poorly defined cell borders. The neoplastic cells are not uncommonly arranged in a lobular pattern associated with fine fibrovascular tissue septae, but they may also be arranged in solid sheets. Brisk mitotic activity, karyorrhexis, and cellular pleomorphism may be prominent features of some tumors. The background of the cellular process is variably composed of neuropil; Homer Wright rosettes may be seen. Also present in some tumors are neuroblasts differentiating towards ganglion cells. Tumor cells are immunoreactive for neuroendocrine markers such as CD56, synaptophysin, chromogranin, and neuron-specific enolase.

When a TWT is evaluated in the pediatric age group, the differential diagnosis includes an intrarenal teratoma (a rare tumor that often displays organogenesis such as skin adnexal structures), and metastatic deposits from extra-renal germ cell tumors with heterologous elements. The differential diagnosis of extra-renal TWT varies depending on the primary location and the sex and age of the patient. For example, for a postmenopausal woman with a TWT presenting in the female reproductive tract, the differential diagnosis may also include primary or metastatic uterine or ovarian carcinosarcoma with heterologous elements. Detailed histological features (identified in properly sampled and well-preserved tumors) combined withimmunohistochemical stains, genetic expression profile studies, and the patient’s age and clinical presentation assist in differentiating these neoplasms from WT or TWT.

Patients with WT are cured by initial therapy in close to 80% of cases. Although different clinical treatment protocols are available in North America (National Wilms’ Tumor Study Group------advocating nephrectomy and pathological diagnosis) and Europe (International Society of Pediatric Oncology/SIOP------advocating initial treatment with neoadjuvant chemotherapy), both protocols are associated with a survival rate of 90%. Similar staging

12

Page 13: 2019 JPIP-A 病理診断生涯教育プログラムLeslie Dodd, MD FCAP Sabrina C. Sopha, MD FCAP Raul Gonzalez, MD FCAP Nichole L. Steidler, MD FCAP Christina Isacson, MD FCAP Justin

metanephric adenoma2

N/C

2090 BRAF V600E WT

CD57

intrarenal neuroblastoma

salt-and-pepper

Homer Wright

CD56

TWT intrarenal teratoma

TWTTWT

WT TWT

WT 80National Wilms’ Tumor Study Group

International Society of Pediatric Oncology/SIOP90 NIH

SIOP

13

Page 14: 2019 JPIP-A 病理診断生涯教育プログラムLeslie Dodd, MD FCAP Sabrina C. Sopha, MD FCAP Raul Gonzalez, MD FCAP Nichole L. Steidler, MD FCAP Christina Isacson, MD FCAP Justin

systems for renal tumors of childhood developed in the US by the National Cancer Institute-sponsored Children’s Oncology Group and in Europe by the SIOP are routinely used for patients with WT. The presence of anaplasia and the identification of some genetic abnormalities have prognostic significance, and a low stage at the time of resection confers a ‘‘favorable’’ prognosis in WTs. TWTs are reported to have a similar overall prognosis as classic WT.

Educational

Questions 1. Which of the following is true regarding the term nephroblastoma with heterologous elements (teratoid Wilms’ tumor)?

6016 It can only be used to designate extrarenal tumors with mature heterologous epithelial and stromal differentiation.

6017 It identifies a subtype of Wilms tumor that is immunonegative for WT1.6018 It is synonymous with fetal rhabdomyomatous nephroblastoma.6019 It is used to designate blastema predominant Wilms tumors.6020 It is used to designate Wilms tumors when the heterologous epithelial and stromal

differentiation comprise >50% of the tumor volume.

2. In cases of Wilms’ tumor, which of the following findings has been shown to be associated with more clinically aggressive disease?

2912 MYCN amplification6021 Diffuse anaplasia6022 t(12;15)p13;q25 chromosomal translocation2028 Homer Wright rosettes5017 BRAF V600E mutation

3. Which of the following genetic alterations can be seen in association with Wilms’ tumor?6023 Loss of imprinting on 11p153613 N-myc amplification6024 Parental isodisomy of chromosome 11p15.56025 t(8;14)6026 t(12;15)(p13;q25)

14

Page 15: 2019 JPIP-A 病理診断生涯教育プログラムLeslie Dodd, MD FCAP Sabrina C. Sopha, MD FCAP Raul Gonzalez, MD FCAP Nichole L. Steidler, MD FCAP Christina Isacson, MD FCAP Justin

WTWT TWT WT

1 31.

(6016)

(6017) WT1(6018)(6019)(6020) 50

2.

(2912) MYCN(6021)(6022) t(12;15) p13;q25 (2028) Homer Wright (5017) BRAF V600E

3. (6023) 11p15(3613) N-myc(6024) 11p15.5(6025) t(8;14)(6026) t(12;15)(p13;q25)

97

15

Page 16: 2019 JPIP-A 病理診断生涯教育プログラムLeslie Dodd, MD FCAP Sabrina C. Sopha, MD FCAP Raul Gonzalez, MD FCAP Nichole L. Steidler, MD FCAP Christina Isacson, MD FCAP Justin

References 1. Argani P, Bruder E, Dehner L, Vujanic GM. Nephroblastic and cystic tumours occurring mainly in children; Nephroblastoma. In: Moch H, Peter A, Ulbright TM, Reuter VE, eds. WHO Classification of Tumours of the Urinary System and Male Genital Organs. Lyon, FR:IARC;2016:44-84.

2. Delahunt B, Grignon DJ, Eble JN. Ch2. Tumors of the kidney. In: Amin MB, Grignon DJ, Srigley JR, Eble JN, eds. Urology Pathology. 1st ed. Philadelphia, PA: Lippincott Williams & Watkins; 2014:126-139.

3. Finn LS, Husain AN. Ch. 17, The Kidney and Lower Urinary Tract. In: Stocker JT, Dehner LP, Husain AN, eds. Stocker & Dehner’s Pediatric Pathology. Fourth ed. Philadelphia, PA: Wolters Kluwer; 2016:840-848.

4. Ghamdi DA, Bakshi N, Akhtar M. Teratoid Wilms Tumor: Report of Three Cases and Review of the Literature. Turk Patoloji Derg. 2017:1-8.

5. Maitra A. Ch. 10, Diseases of Infancy and Childhood. In: Kumar V, Abbas AK, Aster JC, eds. Robbins and Cotran Pathologic Basis of Disease. Ninth ed. Philadelphia: Saunders/Elsevier, 2015:479-481

6. Myers JB, Dall'Era J, Odom LF, McGavran L, Lovell MA, Furness P. Teratoid Wilms' tumor, an important variant of nephroblastoma. J Pediatr Urol. 2007;3(4):282-286.

7. Sultan I, Ajlouni F, Al-Jumaily U, et al. Distinct features of teratoid Wilms tumor. J Pediatr Surg. 2010;45(10):E13-E19.

8. Treetipsatit J, Raveesunthornkiet M, Ruangtrakool R, Sanpaki K, Thorner PS. Teratoid Wilms' tumor: case report of a rare variant that can mimic aggressive biology during chemotherapy. J Pediatr Surg. 2011;46(12):e1-6.

Nilsa C. Ramirez, MDSurgical Pathology CommitteeNationwide Children’s Hospital

Columbus, OH

16

Page 17: 2019 JPIP-A 病理診断生涯教育プログラムLeslie Dodd, MD FCAP Sabrina C. Sopha, MD FCAP Raul Gonzalez, MD FCAP Nichole L. Steidler, MD FCAP Christina Isacson, MD FCAP Justin

1. Argani P, Bruder E, Dehner L, Vujanic GM. Nephroblastic and cystic tumours occurring mainly in children; Nephroblastoma. In: Moch H, Peter A, Ulbright TM, Reuter VE, eds. WHO Classification of Tumours of the Urinary System and Male Genital Organs. Lyon, FR:IARC;2016:44-84.

2. Delahunt B, Grignon DJ, Eble JN. Ch2. Tumors of the kidney. In: Amin MB, Grignon DJ, Srigley JR, Eble JN, eds. Urology Pathology. 1st ed. Philadelphia, PA: Lippincott Williams & Watkins; 2014:126-139.

3. Finn LS, Husain AN. Ch. 17, The Kidney and Lower Urinary Tract. In: Stocker JT, Dehner LP, Husain AN, eds. Stocker & Dehner’s Pediatric Pathology. Fourth ed. Philadelphia, PA: Wolters Kluwer; 2016:840-848.

4. Ghamdi DA, Bakshi N, Akhtar M. Teratoid Wilms Tumor: Report of Three Cases and Review of the Literature. Turk Patoloji Derg. 2017:1-8.

5. Maitra A. Ch. 10, Diseases of Infancy and Childhood. In: Kumar V, Abbas AK, Aster JC, eds. Robbins and Cotran Pathologic Basis of Disease. Ninth ed. Philadelphia: Saunders/Elsevier, 2015:479-481

6. Myers JB, Dall'Era J, Odom LF, McGavran L, Lovell MA, Furness P. Teratoid Wilms' tumor, an important variant of nephroblastoma. J Pediatr Urol. 2007;3(4):282-286.

7. Sultan I, Ajlouni F, Al-Jumaily U, et al. Distinct features of teratoid Wilms tumor. J Pediatr Surg. 2010;45(10):E13-E19.

8. Treetipsatit J, Raveesunthornkiet M, Ruangtrakool R, Sanpaki K, Thorner PS. Teratoid Wilms' tumor: case report of a rare variant that can mimic aggressive biology during chemotherapy. JPediatr Surg. 2011;46(12):e1-6.

Nilsa C. Ramirez, MDSurgical Pathology CommitteeNationwide Children’s Hospital

Columbus, OH

17

Page 18: 2019 JPIP-A 病理診断生涯教育プログラムLeslie Dodd, MD FCAP Sabrina C. Sopha, MD FCAP Raul Gonzalez, MD FCAP Nichole L. Steidler, MD FCAP Christina Isacson, MD FCAP Justin

Case # 2019-02

Diagnosis Fibroma

Site Ovary

Clinical

Summary

A 41-year-old woman presents with abdominal distention, abdominal pain, and increasedurinary frequency. A computed tomography scan shows a 12.0 cm diffusely hypoechoic rightpelvic/ovarian mass. The patient undergoes right salpingo-oophorectomy. Grossly, the tumor is well-circumscribed and firm with lobulated, grey-white, glistening surface. The cut surface of the tumor is white with some areas of hemorrhage.

Master List 2081 Adult granulosa cell tumor1711 Brenner tumor1957 Endometrial stromal sarcoma2540 Fibroma1100 Fibrothecoma1646 Leiomyoma1697 Massive ovarian edema2548 Thecoma

Criteria for

Diagnosis

and

Comments

Histologic sections of the ovarian mass show thin spindle cells with bland, wavy, fusiform nuclei and eosinophilic cytoplasm arranged in a storiform pattern, blending with a variable amount of collagen. Cytological atypia is absent. These findings are consistent with fibroma.

Most tumors of the ovary can be placed into one of 3 major categories: surface epithelialtumors, sex cord-stromal tumors, or germ cell tumors. The sex cord-stromal group includes tumors of mesenchymal and mesonephric origin. Some of these tumors, namely fibromas and thecomas, have a fibrous appearance, and some appear to be derived from granulosa cells or their testicular sex cord counterparts, the Leydig and Sertoli cells.

Ovarian fibromas are the most common benign solid tumors, accounting for an estimated1%-4% of all ovarian tumors. Ovarian fibromas occur most commonly in post-menopausalwomen, with a mean age at diagnosis of late forties and fifties; they rarely occur in theprepubertal age group. Unlike other sex cord-stromal tumors, fibromas are rarely associated with hormone production. Patients are usually asymptomatic, but they can present with abdominal pain, abdominal distention, and increased urinary frequency.

Ovarian fibromas are typically unilateral (syndrome-related tumors are more likely to bebilateral) with an average size of 5.0-6.0 cm. Grossly, ovarian fibromas are spherical, slightly lobulated, encapsulated, grey-white masses covered by a glistening, intact ovarian serosa

18

Page 19: 2019 JPIP-A 病理診断生涯教育プログラムLeslie Dodd, MD FCAP Sabrina C. Sopha, MD FCAP Raul Gonzalez, MD FCAP Nichole L. Steidler, MD FCAP Christina Isacson, MD FCAP Justin

# 2019-02

Fibroma

41 CT 12.0cm

(2081) Adult granulosa cell tumor(1711) Brenner tumor(1957) Endometrial stromal sarcoma(2540) Fibroma(1100) Fibrothecoma(1646) Leiomyoma(1697) Massive ovarian edema(2548) Thecoma

31

1 4

40 50

5.0 6.0cm

19

Page 20: 2019 JPIP-A 病理診断生涯教育プログラムLeslie Dodd, MD FCAP Sabrina C. Sopha, MD FCAP Raul Gonzalez, MD FCAP Nichole L. Steidler, MD FCAP Christina Isacson, MD FCAP Justin

and have a bosselated external surface. Edematous tumors may be soft in consistency and cyst formation is common. The cut surface is grey-white and homogeneous with a whorled pattern and occasional areas of calcification.

Histologically, ovarian fibromas are characterized by fascicles of thin spindle cells with bland, wavy, fusiform nuclei and eosinophilic cytoplasm, blending with a variable amount of collagen. Cytologic atypia is not a prominent feature. About 10% are hypercellular, containing little intercellular collagen. Cellular fibroma (CF) is characterized by bland spindle cells in herringbone pattern with 1 - 3 mitotic figures (MFs) per 10 high-power fields (HPFs). Tumors with more than 4 MFs per 10 HPFs without nuclear atypia are classified as “cellular fibroma of uncertain malignant potential” or “mitotically active cellular fibroma (MACF).” Even typical cellular fibromas can occasionally recur locally, particularly if they are associated with rupture or adherence, and long-term followup for patients with CF and MACF is appropriate. Immunohistochemically, ovarian fibromas are positive for vimentin, inhibin (variable), calretinin, SF1, WT-1, CD34, CD56, SMA, estrogen receptor (ER), and progesterone receptor (PR) and negative for CD10, CD99, desmin, and cytokeratin.

An estimated 1% of ovarian fibromas present as a manifestation of Meigs syndrome, acondition that includes a triad of benign ovarian tumor, ascites, and pleural effusion (usually unilateral and right-sided). Ovarian fibromas can also be associated with Gorlin syndrome (basal cell nevus syndrome), which is characterized by multiple basal cell carcinomas of the skin, odontogenic keratocysts, and ectopic calcifications. Trisomy 12 is a common cytogenetic abnormality associated with benign ovarian fibroma. Ovarian fibromas are often difficult to diagnose preoperatively and are often misdiagnosed as uterine leiomyoma due to the solid nature of the mass on clinical and ultrasound examination. In a minority of patients with ovarian fibroma, the serum level of CA-125 is elevated, which may lead to misdiagnosis of a malignant ovarian tumor. Very rarely, fibromas are associated with peritoneal implants.

The differential diagnosis of fibroma includes thecoma, adult granulosa cell tumor (GCT),Brenner tumor, massive ovarian edema, fibrosarcoma, leiomyoma, and endometrial stromal tumor. Pure thecomas are uncommon and, unlike fibromas, are more likely to present with hormonal manifestations. Grossly, thecomas are unilateral, well-defined, solid, yellow masses. Histology demonstrates spindle cells with moderately pale gray to pink cytoplasm rarely containing lipid droplets and central nuclei. Fibromas and thecomas have similar histomorphologic features and immunoprofiles; many thecomas have regions resembling fibroma and may be labeled fibrothecoma.

Adult GCTs present unilaterally with yellow-tan or gray cut surfaces. Histologically GCTsshow small, bland, cuboidal to polygonal tumor cells with grooved nuclei and scant cytoplasm arranged in various patterns, including Call–Exner bodies (small follicle-like structures filled with acidophilic material), macrofollicular, trabecular, solid, and insular patterns. Fibromas exhibit an individual pericellular reticulin staining pattern, whereas GCTs exhibit a nested pattern of reticulin staining. Additionally, GCTs show a characteristic FOXL2 mutation.

20

Page 21: 2019 JPIP-A 病理診断生涯教育プログラムLeslie Dodd, MD FCAP Sabrina C. Sopha, MD FCAP Raul Gonzalez, MD FCAP Nichole L. Steidler, MD FCAP Christina Isacson, MD FCAP Justin

10 cellular fibroma CF10 HPF 1 3 MF

10HPF 4“cellular fibroma of uncertain malignant potential ”

“mitotically active cellular fibroma MACF ”CF CF MACF

SF1 WT-1 CD34 CD56 SMA ERPR CD10 CD99

1 Meigs3

Gorlin12

CA-125

thecoma adult granulosa cell tumor GCTBrenner tumor massive ovarian edemaendometrial stromal tumor

fibrothecoma

GCT GCTCall-Exner

GCTGCT FOXL2

21

Page 22: 2019 JPIP-A 病理診断生涯教育プログラムLeslie Dodd, MD FCAP Sabrina C. Sopha, MD FCAP Raul Gonzalez, MD FCAP Nichole L. Steidler, MD FCAP Christina Isacson, MD FCAP Justin

Brenner tumors consist of abundant dense fibrous stroma with transitional epithelial nests that show positivity for cytokeratin by immunohistochemistry.

Massive ovarian edema is usually unilateral and demonstrates marked intercellular edemahistologically.

Leiomyomas are lobulated white-grey masses with whorled cut surfaces. Histologydemonstrates spindle cells arranged in a whorled pattern. By immunohistochemistryleiomyomas stain positive for desmin, SMA, and caldesmon, but negative for inhibin.

Low-grade endometrial stromal sarcomas are infiltrative and diffuse proliferations of uniform round or oval cells with abundant small vessels and CD10 and vimentin positivity.

Educational

Questions 1. To which of the following ovarian tumor categories does ovarian fibroma belong?1719 Germ cell tumors1982 Sertoli–Leydig cell tumors1653 Sex cord-stromal tumors6027 Surface epithelial tumors6028 Transitional cell (Brenner) tumors

2. Which of the following syndromes may be associated with ovarian fibroma?6029 Gorlin syndrome (basal cell nevus syndrome) and Carney triad6030 Meigs syndrome and Gorlin syndrome (basal cell nevus syndrome)6031 Meigs syndrome and neurofibromatosis6032 Tuberous sclerosis and Gardner syndrome6033 WAGR syndrome and Beckwith–Wiedemann syndrome

3. Which of the following stains is useful to distinguish fibroma and adult granulosa cell tumor?3734 CD102877 CD345741 GATA-35030 PAX81687 Reticulin

22

Page 23: 2019 JPIP-A 病理診断生涯教育プログラムLeslie Dodd, MD FCAP Sabrina C. Sopha, MD FCAP Raul Gonzalez, MD FCAP Nichole L. Steidler, MD FCAP Christina Isacson, MD FCAP Justin

SMA

Low-grade endometrial stromal sarcomas are infiltrative and diffuse proliferations of uniform

CD10

1 31.

(1719)(1982) -(1653)(6027)(6028)

2.(6029) Gorlin Carney triad(6030) Meigs Gorlin(6031) Meigs(6032) Gardner(6033) WAGR Beckwith–Wiedemann

3.(3734) CD10(2877) CD34(5741) GATA-3(5030) PAX8(1687) Reticulin

97

23

Page 24: 2019 JPIP-A 病理診断生涯教育プログラムLeslie Dodd, MD FCAP Sabrina C. Sopha, MD FCAP Raul Gonzalez, MD FCAP Nichole L. Steidler, MD FCAP Christina Isacson, MD FCAP Justin

References 1. Arya, A, Rao S, Agarwal S, Arora R, Gupta K, Dhawan I. Ovarian fibroma: An unusual morphological presentation with elevated CA-125. Indian J Pathol Microbiol.2008;51(4):523- 524.

2. Chen VW, Ruiz B, Killeen JL, Coté TR, Wu XC, Correa CN. Pathology and classification of ovarian tumors. Cancer. 2003;97(10 Suppl):2631-2642.

3. Kurman RJ, Carcangiu ML, Herrington CS, Young RH, eds. WHO Classification of Tumours of Female Reproductive Organs. 4th Edition. Lyon: IARC. 2014:44-49.

4. Numanoglu C, Kuru O, Sakinci M, Akbayir O, Ulker V. Ovarian fibroma/fibrothecoma: retrospective cohort study shows limited value of risk of malignancy index score. Aust N Z J Obstet Gynaecol. 2013;53(3):287-292.

5. Parwate NK, Patel SM, Arora R, Gupta M. Ovarian fibroma: A clinico-pathological study of 23 cases with review of literature. J Obstet Gynaecol India. 2015;66(6):460-465.

6. Prat J, Scully RE. Cellular fibromas and fibrosarcomas of the ovary: a comparative clinicopathologic analysis of seventeen cases. Cancer. 1981;47:2663-2670.

of the ovary: a case report. Turk J Obstet Gynecol. 2015;1:53-55.8. Zong L, Lin M, Fan X. Mitotically active cellular fibroma of ovary should be differentiated

from fibrosarcoma: a case report and review of literature. Int J Clin Exp Pathol. 2014.7(11):7578-7582.

Archi Patel, MDSurgical Pathology Committee

East Tennessee State UniversityJohnson City, TN

24

Page 25: 2019 JPIP-A 病理診断生涯教育プログラムLeslie Dodd, MD FCAP Sabrina C. Sopha, MD FCAP Raul Gonzalez, MD FCAP Nichole L. Steidler, MD FCAP Christina Isacson, MD FCAP Justin

1. Arya, A, Rao S, Agarwal S, Arora R, Gupta K, Dhawan I. Ovarian fibroma: An unusual morphological presentation with elevated CA-125. Indian J Pathol Microbiol. 2008;51(4):523- 524.

2. Chen VW, Ruiz B, Killeen JL, Coté TR, Wu XC, Correa CN. Pathology and classification of ovarian tumors. Cancer. 2003;97(10 Suppl):2631-2642.

3. Kurman RJ, Carcangiu ML, Herrington CS, Young RH, eds. WHO Classification of Tumours of Female Reproductive Organs. 4th Edition. Lyon: IARC. 2014:44-49.

4. Numanoglu C, Kuru O, Sakinci M, Akbayir O, Ulker V. Ovarian fibroma/fibrothecoma: retrospective cohort study shows limited value of risk of malignancy index score. Aust N Z J Obstet Gynaecol. 2013;53(3):287-292.

5. Parwate NK, Patel SM, Arora R, Gupta M. Ovarian fibroma: A clinico-pathological study of 23 cases with review of literature. J Obstet Gynaecol India. 2015;66(6):460-465.

6. Prat J, Scully RE. Cellular fibromas and fibrosarcomas of the ovary: a comparative clinicopathologic analysis of seventeen cases. Cancer. 1981;47:2663-2670.

7. Yovary: a case report. Turk J Obstet Gynecol. 2015;1:53-55.

8. Zong L, Lin M, Fan X. Mitotically active cellular fibroma of ovary should be differentiated from fibrosarcoma: a case report and review of literature. Int J Clin Exp Pathol. 2014.7(11):7578-7582.

Archi Patel, MDSurgical Pathology Committee

East Tennessee State UniversityJohnson City, TN

25

Page 26: 2019 JPIP-A 病理診断生涯教育プログラムLeslie Dodd, MD FCAP Sabrina C. Sopha, MD FCAP Raul Gonzalez, MD FCAP Nichole L. Steidler, MD FCAP Christina Isacson, MD FCAP Justin

Case # 2019-03

Diagnosis Mitochondrial cardiomyopathy

Site Heart

Clinical

Summary

A 6-month-old baby boy presents to an outside hospital with cardiomyopathy of uncertain etiology. The baby is transferred to a tertiary care center where cardiac transplantation is performed. Representative sections of the explanted heart are submitted. PAS stains with and without diastase show no increase in intracytoplasmic glycogen or increase of PAS-positive diastase resistant material. A modified Gomori trichrome stain shows granular staining of the cytoplasm. Diagnostic electron microscopy reveals increased numbers of an abnormal organelle.

Master List 5598 Arrhythmogenic cardiomyopathy5599 Glycogen storage disorder5600 Mitochondrial cardiomyopathy5601 Mucopolysaccharidoses1122 Myocarditis

Criteria for

Diagnosis

and

Comments

H&E sections of the heart show myocytes with reactive nuclear changes characterized by mild pleomorphism, hyperchromasia, and enlargement, and prominent vacuolar cytoplasmic change. Intracytoplasmic membrane-bound vacuoles are absent. There is no evidence of myocyte disarray or inflammation.

PAS stains with and without diastase show no increase in intracytoplasmic glycogen or diastase-resistant material. A modified Gomori trichrome stain shows granular staining of the cytoplasm correlating with abnormal mitochondria identified by electron microscopy. A toluidine blue stain highlights distended and displaced myofilaments within the cytoplasm of the myocytes.

Electron microscopy reveals increased numbers of mitochondria within the cytoplasm. Many of the mitochondria are dysmorphic, consisting of circular, concentrically (onion-like) arranged cristae and some arranged in parallel arrays. No abnormal metabolic product is identified. The overall changes are diagnostic of mitochondrial cardiomyopathy. The etiology of the mitochondrial cardiomyopathy cannot be determined from the morphologic features alone.

Cardiomyopathy (CMP) is a rare disorder in children with an estimated 1.3 cases annually per 100,000 in children less than 10 years of age. It is associated with high morbidity and mortality. Recent studies have found up to 26% of cases of pediatric cardiomyopathy to be due to inborn errors of metabolism, a subset of which fall into the category of mitochondrial disorders. Mitochondrial diseases are a clinically, genetically, and biochemically heterogeneous group of disorders. Mitochondria are important providers of energy as they

26

Page 27: 2019 JPIP-A 病理診断生涯教育プログラムLeslie Dodd, MD FCAP Sabrina C. Sopha, MD FCAP Raul Gonzalez, MD FCAP Nichole L. Steidler, MD FCAP Christina Isacson, MD FCAP Justin

# 2019-03

Mitochondrial cardiomyopathy

6PAS PAS

PAS

(5598) Arrhythmogenic cardiomyopathy(5599) Glycogen storage disorder(5600) Mitochondrial cardiomyopathy(5601) Mucopolysaccharidoses(1122) Myocarditis

HE

PAS PAS

cardiomyopathy CMP 10 10 1.326

electron transport chain ETCATP

27

Page 28: 2019 JPIP-A 病理診断生涯教育プログラムLeslie Dodd, MD FCAP Sabrina C. Sopha, MD FCAP Raul Gonzalez, MD FCAP Nichole L. Steidler, MD FCAP Christina Isacson, MD FCAP Justin

generate adenosine triphosphate via the electron transport chain (ETC) and oxidative phosphorylation system. Disorders of mitochondria, therefore, preferentially affect organ systems with high energy requirements such as the heart, brain, and neuromuscular systems. Mitochondrial disorders may be primary or secondary. Primary disorders are due to mutations in mitochondrial DNA or nuclear DNA. Most mitochondrial myopathies are non-Mendelian, since mitochondrial DNA is passed maternally with the cytoplasm and not in the nucleus of dividing cells. Secondary mutations are the result of an insult affecting mitochondrial function either directly or indirectly or by damaging the mitochondrial genome. Inadequate levels of mitochondrial DNA and its mutations can cause defects in the synthesis of key subunits ofETC complexes.

The clinical presentation of mitochondrial myopathy is highly variable. Muscle fatigue and weakness, which can manifest in infants as failure to thrive, are frequent symptoms and are easily overlooked. There is great clinical variability in presentation, even for the same mutations, making diagnosis difficult. The prevalence of inherited mitochondrial disease has been estimated at 1 in 5,000. Cardiomyopathy develops in 20%-40% of children with mitochondrial disease.

The morphologic features of the heart in mitochondrial CMP are not specific, leading to a broad differential diagnosis including storage, metabolic, genetic, and environmental disorders. Hypertrophic CMP is the most common manifestation of mitochondrial CMP; however, dilated CMP, restrictive CMP, histiocytoid CMP, or left ventricular hypertrabeculation/noncompaction (LVHT) CMP may also occur. LVHT is characterized by prominent ventricular trabeculations or channels that extend into the myocardium, creating deep recesses within the myocardium. Mitochondrial diseases may be hereditary or sporadic. Mitochondrial myopathy, encephalopathy, lactic acidosis, and stroke-like episodes (MELAS), myoclonic epilepsy with ragged red fibers (MERRF), chronic progressive external ophthalmoplegia, Leigh syndrome, Xlinked cardiomyopathy, mitochondrial myopathy, cyclic neutropenia (Barth syndrome), and neuropathy, ataxia, and retinitis pigmentosa (NARP) are examples of syndromic mitochondrial disorders causing hypertrophic CMP. Skeletal muscle biopsy is considered the gold standard for the diagnosis of mitochondrial disorders. Ragged red fibers are considered the histologic hallmark of mitochondrial dysfunction, but these are not typically seen in children.

Microscopic features of mitochondrial myopathy vary. Fusiform enlargement of affected myocytes around the perinuclear region with cytoplasmic clearing and replacement of cross striae by fine granules may suggest a mitochondrial abnormality. Excessively granular eosinophilic cytoplasm of hepatocytes, cardiac myocytes, skeletal muscle, and proximal renal tubules also may indicate mitochondrial hyperplasia.

Laboratory workup includes analysis for lactate, pyruvate, phosphocreatine, amino acid abnormalities, organic acids, creatinine, and quantification of coenzyme Q. The diagnosis may be made by special stains including cytochrome oxidase (oxidasenegative myocytes), mitochondrial antigens that can demonstrate mitochondrial hyperplasia, and staining for succinate dehydrogenase. Postmortem analysis of urine and vitreous fluid, frozen tissue, and a fibroblast culture provide a comprehensive study. Genomic and/or mitochondrial sequencing is increasingly being used for diagnosis. Ultrastructural analysis will show

28

Page 29: 2019 JPIP-A 病理診断生涯教育プログラムLeslie Dodd, MD FCAP Sabrina C. Sopha, MD FCAP Raul Gonzalez, MD FCAP Nichole L. Steidler, MD FCAP Christina Isacson, MD FCAP Justin

DNADNA DNA

DNA ETC

5,000 120 40

LVHT LVHT

MELAS MERRFragged red fiber CPEO Leigh

Barth X NARP

ragged red fiber

Q

29

Page 30: 2019 JPIP-A 病理診断生涯教育プログラムLeslie Dodd, MD FCAP Sabrina C. Sopha, MD FCAP Raul Gonzalez, MD FCAP Nichole L. Steidler, MD FCAP Christina Isacson, MD FCAP Justin

paracrystalline inclusions, enlarged mitochondria, and concentric onion-like cristae.

The differential diagnosis of mitochondrial myopathy includes glycogen storage diseases, mucopolysaccharidoses, myocarditis, and arrhythmogenic cardiomyopathy. Glycogen storage diseases (GSD) are inherited disorders due to a defect in enzymes that are involved in glycogen metabolism. The 2 types most commonly affecting the heart are type IIa and IIb GSD. Infants with type IIa (Pompe disease), which is due to deficiency of alpha-1,4, alpha 1-6glucosidase (acid maltase), manifest disease in infantile or late forms depending on the severity of the enzymatic defect. There is left ventricular hypertrophy. Grossly, the heart can weigh 3-10 times the normal size and show a globular configuration. Microscopically the myocytes are markedly distended with vacuolated and lacey cytoplasm due to the accumulation of PAS-positive glycogen. Ultrastructural examination shows the glycogen to be at least partially membrane bound. Type IIb (Danon disease) is an Xlinked disease caused bya deficiency of lysosome-associated membrane protein-2 (LAMP-2). Affected children present with hypertrophic CMP, muscle weakness, and mental retardation (70%). Microscopic examination reveals a vacuolar myopathy. The myocytes contain PAS- and acid phosphatase-positive inclusions. Ultrastructural examination shows intracytoplasmic membrane-bound vacuoles containing glycogen.

Mucopolysaccharidoses are lysosomal storage disorders that commonly affect the heart manifested by valvular insufficiency. Irregular nodular deposits are present along the valves. There is intimal plaque formation in the aorta and other systemic vessels. Microscopy reveals vacuolated cells containing acid mucopolysaccharides and glycolipids. Ultrastructurally, membrane-bound vacuoles contain concentric and parallel lamellae.

Myocarditis, or inflammatory CMP, is an inflammatory disease of the myocardium and can be infectious, autoimmune, allergic, or drug induced and is classified under the acquired forms of cardiomyopathy. It is diagnosed by endomyocardial biopsy, which shows inflammation with or without myocyte necrosis. It can be further subdivided according to the type of inflammatory cell present: neutrophils, eosinophils, or lymphocytes. It can resolve or progress to dilated CMP. The most common cause of inflammatory CMP in the United States is a lymphocyticmyocarditis due to a virus, typically Coxsackie viruses A and B or another enterovirus.

Arrhythmogenic CMP is an autosomal dominant disorder that manifests itself with right-sided heart failure and rhythm abnormalities. Morphologically, the right ventricular wall is thinned with fatty infiltration and mild fibrosis. Our understanding of the underlying genetic causes of arrhythmogenic CMP is evolving with next generation sequencing and other molecular and genetic analyses. Mutations in genes involved in desmosomal junctional proteins are well recognized to be associated with the development of arrhythmogenic CMP. Recently mutations involving nondesmosomal genes, such as mutations in transforming growth factorbeta, have also been identified in families with a history of arrhythmogenic CMP.

30

Page 31: 2019 JPIP-A 病理診断生涯教育プログラムLeslie Dodd, MD FCAP Sabrina C. Sopha, MD FCAP Raul Gonzalez, MD FCAP Nichole L. Steidler, MD FCAP Christina Isacson, MD FCAP Justin

arrhythmogenic cardiomyopathy 2

IIa IIbIIa Pompe

3 10PAS

IIb Danon2 LAMP-2 X

70PAS

mucopolysaccharidosis

myocarditis

A B

TGF-

31

Page 32: 2019 JPIP-A 病理診断生涯教育プログラムLeslie Dodd, MD FCAP Sabrina C. Sopha, MD FCAP Raul Gonzalez, MD FCAP Nichole L. Steidler, MD FCAP Christina Isacson, MD FCAP Justin

Educational

Questions

1. A mother brings her child to the pediatrician stating that the child is easily tired. The child exhibits developmental delay. Cardiac examination reveals a hypertrophic cardiomyopathy. A cardiac biopsy shows vacuolated myocytes that are PAS positive. Acid maltase activity levels were normal. Which of the following is the best diagnosis?

6034 A mitochondrial disorder6035 Arrhythmogenic cardiomyopathy6036 Type IIa glycogen storage diseases (Pompe disease)6037 Type IIb glycogen storage disease (Danon disease)6038 Viral myocarditis

2. Which of the following most accurately describes mitochondrial disorders?6039 Morphologically, the right ventricle is thinned with fatty infiltration and fibrosis.6040 Ragged red fibers are not associated with mitochondrial cardiomyopathy.6041 Stains for cytochrome oxidase and mitochondrial antigens are useful in making the diagnosis.6042 They usually present with dilated cardiomyopathy.6043 They are easily recognized disorders due to the specific symptomatology.

3. Which of the following is the most frequent pattern of inherited mitochondrial myopathies?6044 Autosomal dominant6045 Autosomal recessive6046 Non-Mendelian6047 X-linked dominant6048 X-linked recessive

References 1. Byers SL, Ficicioglu C. Infant with cardiomyopathy: when to suspect inborn errors of metabolism? World J Cardiol. 2014;6(11):1149-1155.

2. Dewulf JP, Barrea C, Vincent M-F, et al. Evidence of a wide spectrum of cardiac involvement due to ACAD9 mutations: report on nine patients. Molecular Genetics and Metabolism. 2016; 118(3):185-189.

3. Karnouch J, Protonotarios A, Syrris P. Genetic basis of arrhythmogenic cardiomyopathy. Curr Opin Cardiol. 2018;33(3):276-281.

4. Meyers DE, Basha HI, Koenig MK. Mitochondrial cardiomyopathy: pathophysiology, diagnosis, management. Tex Heart Inst J. 2013;40(4):385-394.

5. Sweeney RT, Davis GJ, Noonan JA. Cardiomyopathy of unknown etiology: Barth Syndrome unrecognized. Congenit Heart Dis. 2008;3:443-448.

6. Towbin JA. Inherited cardiomyopathies. Circ J. 2014;78(10):2347-2356.7. Wallis G, Fricker FJ. Neonatal cardiomyopathy. NeoReviews. 2012;13:e711-e721.

Deborah A. Belchis, MDSurgical Pathology Committee

Johns Hopkins Medical InstitutionBaltimore, Maryland

32

Page 33: 2019 JPIP-A 病理診断生涯教育プログラムLeslie Dodd, MD FCAP Sabrina C. Sopha, MD FCAP Raul Gonzalez, MD FCAP Nichole L. Steidler, MD FCAP Christina Isacson, MD FCAP Justin

1 31.

PAS

(6034)(6035)(6036) IIa Pompe(6037) IIb Danon(6038)

2.(6039)(6040) ragged red fibers(6041)(6042)(6043)

3. (6044)(6045)(6046) (6047) X(6048) X

97

1. Byers SL, Ficicioglu C. Infant with cardiomyopathy: when to suspect inborn errors of metabolism? World J Cardiol. 2014;6(11):1149-1155.

2. Dewulf JP, Barrea C, Vincent M-F, et al. Evidence of a wide spectrum of cardiac involvement due to ACAD9 mutations: report on nine patients. Molecular Genetics and Metabolism. 2016; 118(3):185-189.

3. Karnouch J, Protonotarios A, Syrris P. Genetic basis of arrhythmogenic cardiomyopathy. Curr Opin Cardiol. 2018;33(3):276-281.

4. Meyers DE, Basha HI, Koenig MK. Mitochondrial cardiomyopathy: pathophysiology, diagnosis, management. Tex Heart Inst J. 2013;40(4):385-394.

5. Sweeney RT, Davis GJ, Noonan JA. Cardiomyopathy of unknown etiology: Barth Syndrome unrecognized. Congenit Heart Dis. 2008;3:443-448.

6. Towbin JA. Inherited cardiomyopathies. Circ J. 2014;78(10):2347-2356.7. Wallis G, Fricker FJ. Neonatal cardiomyopathy. NeoReviews. 2012;13:e711-e721.

Deborah A. Belchis, MDSurgical Pathology Committee

Johns Hopkins Medical InstitutionBaltimore, Maryland

33

Page 34: 2019 JPIP-A 病理診断生涯教育プログラムLeslie Dodd, MD FCAP Sabrina C. Sopha, MD FCAP Raul Gonzalez, MD FCAP Nichole L. Steidler, MD FCAP Christina Isacson, MD FCAP Justin

Case # 2019-04

Diagnosis TFE3-associated renal cell carcinoma

Site Kidney

Clinical

Summary

A 50-year-old man presents with longstanding hematuria and flank pain, and a 9.5 x 8.5 x 6.8 cm right renal mass with retroperitoneal lymphadenopathy is identified on computed tomography scan. The patient undergoes radical nephrectomy, where a similarly-sized white-tan firm renal mass is found invading the pelvicalyceal system, renal sinus, and perinephric tissue, with a renal vein thrombosis. A total of 34 of 45 regional lymph nodes are involved by metastatic tumor. The adrenal gland is present and uninvolved. On immunohistochemistry, the tumor cells are negative for multiple keratins and melanocytic markers. Tumor cells mark with CD10, P504S, TFE3, and CA-IX

Master List 2632 Clear cell papillary renal cell carcinoma1665 Clear cell renal cell carcinoma5602 Multilocular cystic renal cell neoplasm of low malignant potential1797 Papillary renal cell carcinoma5603 TFE3-associated renal cell carcinoma

Criteria for

Diagnosis

and

Comments

Histologic sections show a fairly well-defined, non-encapsulated renal mass with variably solid/nested and papillary architecture with focal necrosis and hemorrhage. Tumor cells have discrete/defined cell borders and abundant clear to eosinophilic flocculent cytoplasm. Nuclei are relatively uniform, with vesicular chromatin and nucleoli visible at 10x. Psammoma bodies are variably present.

Based on the morphology and immunoprofile, this tumor is classified as a TFE3-associated renal cell carcinoma (RCC). The Microphthalmia-TFE (MiT) translocation renal cell carcinoma family is composed of 2 members: TFE3-associated RCC (involving Xp11 translocations) and TFEB-associated RCC (involving t(6;11)). The TFE3-associated RCCs were first recognized in the 2004 World Health Organization (WHO) classification as Xp11 translocation RCC. They were reclassified in the 2016 WHO classification along with t(6;11) RCCs under the broader heading of MiT family translocation RCC. Approximately 20% - 75% of pediatric RCCs are TFE3-associated RCCs. Only 1.6%-4% of adult RCCs are TFEB-associated and fewer than 100 cases have been reported in the literature.

TFE3-associated RCCs harbor fusions of TFE3 with a variety of gene partners, all of which act as promoters to upregulate TFE3 expression from constitutively low levels to levels detectable by immunohistochemistry. The most common partner is PRCC, resulting from t(X;1)(p11.3;q21), which was first described in 1986 and is the first known example of translocation-associated RCC. The second-most common partner is ASPL (also called

34

Page 35: 2019 JPIP-A 病理診断生涯教育プログラムLeslie Dodd, MD FCAP Sabrina C. Sopha, MD FCAP Raul Gonzalez, MD FCAP Nichole L. Steidler, MD FCAP Christina Isacson, MD FCAP Justin

# 2019-04

TFE3 TFE3-associated renal cell carcinoma

50 CT 9.5 x 8.5 x 6.8cm

45 34CD10 P504S TFE3

CA-IX

(2632) Clear cell papillary renal cell carcinoma(1665) Clear cell renal cell carcinoma(5602) Multilocular cystic renal cell neoplasm of low malignant potential(1797) Papillary renal cell carcinoma(5603) TFE3 TFE3-associated renal cell carcinoma

10

TFE3 renal cell carcinoma , RCC MiT TFE3 RCC Xp11

TFEB RCC t(6;11) TFE3 RCC 2004 WHOXp11 RCC 2016 WHO t(6;11)

RCC MiT RCC RCC 20 75TFE3 RCC RCC 1.6 4 TFEB

100

TFE3 RCC TFE3 TFE3

PRCC 1986t(X;1)(p11.3;q21) RCC 2 1991

t(X;17)(p11.2;q25) ASPL ASPSCR1 RCC 2

35

Page 36: 2019 JPIP-A 病理診断生涯教育プログラムLeslie Dodd, MD FCAP Sabrina C. Sopha, MD FCAP Raul Gonzalez, MD FCAP Nichole L. Steidler, MD FCAP Christina Isacson, MD FCAP Justin

ASPSCR1), resulting from t(X;17)(p11.2;q25), the second known case of translocation RCC, first reported in 1991. Of note, this is the same fusion gene in alveolar soft part sarcoma. Other recurrent partners include NONO, SFPQ, CLTC, and RBM10. TFEB-associated RCCs result from t(6;11), which fuses TFEB with MALAT1, a gene which is transcribed into a non-coding ribonucleic acid of unknown function. When overexpressed, TFE3 and TFEB each act as aberrant transcription factors that activate a variety of downstream targets, many of which are normally activated by other MiT family members (such as cathepsin K).

Microscopically, TFE3-associated RCCs are composed of papillary or nested epithelioid tumor cells with ample clear to eosinophilic cytoplasm, distinct cell borders, and abundant psammoma bodies. The morphology of PRCC-TFE3 RCCs differs from ASPL-TFE3 RCCs: PRCC fusions always show a nested (solid, alveolar, acinar, or tubular) pattern with foci of papillary architecture with less abundant cytoplasm and fewer psammoma bodies as compared with ASPL. However, these morphologic differences are not sufficiently distinct to rely on H&E assessment alone for genetic classification. Some TFE3-associated RCCs show melanin pigment (the so-called “melanotic translocation RCC,” including fusion partners SFPQ and ARID1B), creating overlap between TFE3-associated RCC and TFE3-associated PEComas. The SFPQ-TFE3 tumors have a distinctive morphology of subnuclear vacuoles and nuclear palisading, similar to that seen in endometrium. Overall, TFE3-associated RCCs show high-grade features (necrosis and renal parenchymal invasion). Depending on the elements present, TFE3-associated tumors may mimic clear cell RCC, papillary RCC, clear cell papillary RCC, or multilocular cystic renal neoplasm of low malignant potential. TFEB-associated RCCs, in contrast, are biphasic with nests of larger periepithelioid cells and smaller cells clustered around basement membrane material. Entrapped single renal tubules are found at the tumor edge.

In contrast to other RCCs, the MiT family of translocation-associated RCCs under expresses cytokeratins and epithelial membrane antigens but still consistently expresses PAX8 and other markers of renal tubular origin. The melanotic translocation RCC subgroup expresses melanocytic markers and will show intracytoplasmic melanin pigment. The t(6;11) tumors consistently express Melan-A, HMB45 and cathepsin K, which are also expressed in TFE3-associated RCCs to a lesser degree (60% labeling for cathepsin K, which is likely dependent on the fusion partner, with PRCC-TFE3 fusion tumors having higher rates of expression than ASPL-TFE3 fusion tumors). Diagnosis is by reverse transcriptase-polymerase chain reaction (PCR), immunohistochemistry using an antibody to the C-terminus of TFE3 or an antibody for the TFEB protein, or fluorescence in situ hybridization (FISH) using TFE3 or TFEB break-apart assays, which are the preferred methods as FISH is less susceptible to fixation issues and antibody incubation times.

Survival for patients with TFE3-associated RCCs is comparable to those with clear cell RCCs and significantly worse than that for patients with papillary RCC. Within the TFE3-associated RCCs, however, there remains tremendous variability in survival rates, which is hypothesized to be related to the fusion partner. Fusions with ASPL result in tumors with more regional lymph node metastasis; however, most of these node-positive patients remain disease-free

36

Page 37: 2019 JPIP-A 病理診断生涯教育プログラムLeslie Dodd, MD FCAP Sabrina C. Sopha, MD FCAP Raul Gonzalez, MD FCAP Nichole L. Steidler, MD FCAP Christina Isacson, MD FCAP Justin

NONO SFPQ CLTC RBM10 TFEB RCC t(6;11) TFEB MALAT1

TFE3 TFEBMiT K

TFE3 RCCPRCC-TFE3

RCC ASPL-TFE3 RCC PRCCASPL

HETFE3 RCC

melanotic translocation RCC SFPQ ARID1BTFE3 RCC TFE3 PEComa SFPQ-TFE3

TFE3 RCCTFE3 clear cell RCC papillary RCC

clear cell papillary RCC multilocular cystic renal neoplasm of low malignant potential TFEB RCC

RCC MiT EMAPAX8 RCC

t(6;11) Melan-A HMB45K TFE3 RCC K

60 PRCC-TFE3ASPL-TFE3 RT-PCR TFE3 C

TFEB TFE3 TFEB break-apart assayFISH FISH

TFE3 RCC RCC RCCTFE3 RCC

ASPLTFEB

RCC TFE3 20 30

37

Page 38: 2019 JPIP-A 病理診断生涯教育プログラムLeslie Dodd, MD FCAP Sabrina C. Sopha, MD FCAP Raul Gonzalez, MD FCAP Nichole L. Steidler, MD FCAP Christina Isacson, MD FCAP Justin

without adjuvant therapy. TFEBassociated RCCs are considered more indolent than the TFE3-associated tumors, with a potential for late metastasis 20-30 years after initial diagnosis.

Clear cell renal cell carcinoma (ccRCC) is a morphologically heterogeneous group accounting for 65%-70% of all renal cancers. While the majority of ccRCCs occur sporadically, several familial syndromes give rise to them, including von Hippel-Lindau, Cowden, and Birt-Hogg-Dube syndromes, tuberous sclerosis, and succinate dehydrogenase-deficient renal cell carcinoma syndrome. In sporadic and VHL cases alike, these tumors have a classic vascular pattern related to inactivation of VHL on 3p25 and upregulation of hypoxia-inducible factor alpha. ccRCCs are architecturally diverse and include solid alveolar patterns, most commonly with a network of delicate vessels. Tumor cell cytoplasm is filled with lipids andglycogen which dissolve during processing, resulting in the so-called “clear cell” appearance, but many show eosinophilic cytoplasm, particularly high-grade tumors and areas adjacent to necrosis or hemorrhage. ccRCCs may express AE1/3, CAM5.2, and/or EMA, but CK7 expression is rare and limited. ccRCCs also express PAX8 and CA-IX. CD10 and vimentin are also expressed but are nonspecific.

Multilocular cystic renal neoplasm of low malignant potential (formerly multilocular cystic renal cell carcinoma) accounts for fewer than 1% of all renal tumors and is usually discovered incidentally. Tumors consist of variably-sized cysts separated by thin septa containing scattered tumor cells. Multilocular cystic renal neoplasm of low malignant potential is not a diagnostic consideration once mural nodules of tumor are identified. Tumor cells have ample clear cytoplasm with International Society of Urologic Pathology (ISUP) grade 1 to 2 nucleoli and are found lining the cyst walls and in small quantities within the septae. Necrosis, vascular invasion, and sarcomatoid features are also incompatible with the diagnosis. VHL mutations have been found in 25% of tumors, and chromosome 3p deletion has been found in 74%. Using strict criteria, no tumor has ever been found to recur or metastasize.

Papillary renal cell carcinoma (PRCC) is the second-most common RCC after ccRCC, with a wide age range. In the pediatric group, PRCCs are proportionately more common than in adults, where it is associated with end-stage renal disease or acquired cystic renal disease. There is a familial PRCC syndrome with a high degree of penetrance, and tumors are also part of the Birt-Hogg-Dube syndrome. PRCC is more likely than ccRCC to show necrosis and undergo spontaneous hemorrhage, and up to 30% of cases show calcifications. Morphologically, tumors are wellcircumscribed and composed of papillae formed by delicate fibrovascular cores with foamy macrophages and psammoma bodies. Morphology is variable, from tubular to a packed, solid appearance. Tumors have historically been divided into Type I and Type II tumors, and although Type II tumors may actually represent a heterogeneousgroup, the designation can still be helpful. Type I tumors have papillae covered with a single layer of cells with scant, pale cytoplasm, while Type II tumors show nuclear stratification and higher nuclear grade, with more eosinophilic cells. A third category, oncocytoma-like PRCC, has not been fully characterized. Tumor cells express AE1/3, CAM 5.2, HMWK, EMA, and others, similar to ccRCC. CK7 expression is more common in Type I PRCCs than Type II.

38

Page 39: 2019 JPIP-A 病理診断生涯教育プログラムLeslie Dodd, MD FCAP Sabrina C. Sopha, MD FCAP Raul Gonzalez, MD FCAP Nichole L. Steidler, MD FCAP Christina Isacson, MD FCAP Justin

ccRCC 65 70ccRCC von Hippel-Lindau Cowden Birt-Hogg-Dube

VHL 3p25 VHLccRCC

ccRCC AE1/3 CAM5.2 EMA CK7ccRCCs PAX8 CA-IX CD10

multilocular cystic renal neoplasm of low malignant potentialmultilocular cystic renal cell carcinoma 1

International Society of Urologic Pathology ISUP

VHL 253p 74

PRCC ccRCC 2 RCC PRCC

PRCC Birt-Hogg-Dube PRCC ccRCC 30

I IIII

I II3

PRCC ccRCC AE1/3 CAM 5.2 HMWK EMA CK7 I PRCC II

7 17 YccRCC

39

Page 40: 2019 JPIP-A 病理診断生涯教育プログラムLeslie Dodd, MD FCAP Sabrina C. Sopha, MD FCAP Raul Gonzalez, MD FCAP Nichole L. Steidler, MD FCAP Christina Isacson, MD FCAP Justin

Tumors show trisomy/tetrasomy 7 and trisomy 17, and loss of Y. Spread is similar to ccRCC(vascular).

Clear cell papillary renal cell carcinoma is an indolent tumor accounting for 1%-4% of all renal tumors. It arises in the setting of end-stage renal disease and von Hippel-Lindau syndrome, but is most frequently detected incidentally. Tumor morphology is variable, with tubular, papillary, acinar, cystic, ribbon-like, and solid patterns. Other patterns include a collapsed acinar-like pattern, or solid pattern. The vascular network is delicate, similar to ccRCC. Tumor cells are cuboidal to low-columnar, with characteristic uniform palisading ISUP grade 1 to 2 nuclei oriented away from the basal aspect of the cells. Apical snouts may be seen. Tumor necrosis, perirenal invasion, and lymphovascular invasion are absent. Tumor cells express CK7 diffusely and mark with PAX2 and PAX8. CA-IX characteristically stains cells in a socalled “cup-like” pattern.

Educational

Questions

1. Which of the following statements is true regarding TFE3-associated renal cell carcinomas (RCCs)?

6049 Generally speaking, TFE3 RCCs do not have necrosis or other high-grade features.6050 TFE3 RCCs are less aggressive than TFEB RCCs.6051 TFE3 RCCs may mimic clear cell RCCs.6052 TFE3 RCCs show morphologic, immunophenotypic, and behavioral uniformity.6053 TFE3 RCC spread is primarily lymphatic.

2. Which of the following statements is true regarding TFE3 fusion tumors?6054 ASPL fusion tumors are part of the melanotic translocation RCC family.6055 ASPL fusion tumors show fewer regional lymph node metastases than PRCC fusion

tumors.6056 PRCC fusion tumors show more psammoma bodies than ASPL fusion tumors.6057 PRCC fusion tumors show higher rates of cathepsin K expression than ASPL fusion

tumors.6058 TFE3 fusion partners include PRCC, ASPL, SFPQ, and MALAT1.

3. Which of the following statements is true regarding ancillary testing for TFE3-associated tumors (immunohistochemistry, polymerase chain reaction [PCR], and fluorescence in situ hybridization [FISH])?

6059 FISH using a TFE3 fusion probe is the preferred test due to lack of issues with fixation and/or incubation times.

6060 Reverse transcriptase PCR is an unreliable method of diagnosing TFE3 RCCs.6061 TFE3 immunophenotype is variable and depends on the fusion partner.6062 TFE3 immunostain consistently detects expression in TFE3 tumors and is the test of

choice for diagnosis.6063 TFE3 RCCs consistently express Melan A, HMB45, and cathepsin K.

40

Page 41: 2019 JPIP-A 病理診断生涯教育プログラムLeslie Dodd, MD FCAP Sabrina C. Sopha, MD FCAP Raul Gonzalez, MD FCAP Nichole L. Steidler, MD FCAP Christina Isacson, MD FCAP Justin

clear cell papillary renal cell carcinoma 1 4von Hippel-Lindau

ccRCCISUP

Apical snoutsCK7

PAX2 PAX8 CA-IX

1 31. TFE3 RCC

(6049) TFE3 RCC(6050) TFE3 RCC TFEB RCC(6051) TFE3 RCC RCC(6052) TFE 3 RCC(6053) TFE3 RCC

2. TFE3(6054) ASPL RCC(6055) ASPL PRCC(6056) PRCC ASPL(6057) PRCC ASPL K(6058) TFE3 PRCC ASPL SFPQ MALAT1

3. TFE3 PCRFISH

(6059) TFE3 FISH

(6060) RT-PCR TFE3 RCC(6061) TFE3(6062) TFE3 TFE3(6063) TFE3 RCC Melan A HMB45 K

97

41

Page 42: 2019 JPIP-A 病理診断生涯教育プログラムLeslie Dodd, MD FCAP Sabrina C. Sopha, MD FCAP Raul Gonzalez, MD FCAP Nichole L. Steidler, MD FCAP Christina Isacson, MD FCAP Justin

References 1. Antic T, Taxy JB, Alikhan M, Segal J. Melanotic Translocation Renal Cell Carcinoma with a Novel ARID1B-TFE3 Gene Fusion.” Am J Surg Path. 2017;41(11):1576-1580.

2. Moch H, Humphrey PA, Ulbright TM, Reuter VE, eds. WHO Classification of Tumours of the Urinary System and Male Genital Organs. Lyon, FR: International Agency for Research on Cancer. 2016;18-40.

3. Want XT, Xia QY, Ni H, et al. SFPQ/PSF-TFE3 renal cell carcinoma: a clinicopathologic study emphasizing extended morphology and reviewing the differences between SFPQ-TFE3 RCC and the corresponding mesenchymal neoplasm despite an identical gene fusion. Hum Path. 2017;63:190-200.

4. Xia QY, Wang Z, Chen N, et al. Xp11.2 translocation renal cell carcinoma with NONO-TFE3 gene fusion: morphology, prognosis, and potential pitfall in detecting TFE3 gene rearrangement. Mod Path. 2017; 30(3):416-426.

5. Xiong L, Chen X, Liu N, et al. PRCC-TFE3 dual-fusion FISH assay: A new method for identifying PRCC-TFE3 renal cell carcinoma in paraffinembedded tissue. PLoS, published online 2017. https://doi.org/10.1371/journal.pone.0185337 Accessed November 8, 2018.

Sabrina C. Sopha, MD, FCAPSurgical Pathology Committee

University of Maryland Baltimore Washington Medical CenterGlen Burnie, Maryland

42

Page 43: 2019 JPIP-A 病理診断生涯教育プログラムLeslie Dodd, MD FCAP Sabrina C. Sopha, MD FCAP Raul Gonzalez, MD FCAP Nichole L. Steidler, MD FCAP Christina Isacson, MD FCAP Justin

1. Antic T, Taxy JB, Alikhan M, Segal J. Melanotic Translocation Renal Cell Carcinoma with a Novel ARID1B-TFE3 Gene Fusion.” Am J Surg Path. 2017;41(11):1576-1580.

2. Moch H, Humphrey PA, Ulbright TM, Reuter VE, eds. WHO Classification of Tumours of the Urinary System and Male Genital Organs. Lyon, FR: International Agency for Research on Cancer. 2016;18-40.

3. Want XT, Xia QY, Ni H, et al. SFPQ/PSF-TFE3 renal cell carcinoma: a clinicopathologic study emphasizing extended morphology and reviewing the differences between SFPQ-TFE3 RCC and the corresponding mesenchymal neoplasm despite an identical gene fusion. Hum Path.2017;63:190-200.

4. Xia QY, Wang Z, Chen N, et al. Xp11.2 translocation renal cell carcinoma with NONO-TFE3 gene fusion: morphology, prognosis, and potential pitfall in detecting TFE3 gene rearrangement. Mod Path. 2017; 30(3):416-426.

5. Xiong L, Chen X, Liu N, et al. PRCC-TFE3 dual-fusion FISH assay: A new method for identifying PRCC-TFE3 renal cell carcinoma in paraffinembedded tissue. PLoS, published online 2017. https://doi.org/10.1371/journal.pone.0185337 Accessed November 8, 2018.

Sabrina C. Sopha, MD, FCAPSurgical Pathology Committee

University of Maryland Baltimore Washington Medical CenterGlen Burnie, Maryland

43

Page 44: 2019 JPIP-A 病理診断生涯教育プログラムLeslie Dodd, MD FCAP Sabrina C. Sopha, MD FCAP Raul Gonzalez, MD FCAP Nichole L. Steidler, MD FCAP Christina Isacson, MD FCAP Justin

Case # 2019-05

Diagnosis Hepatocellular carcinoma

Site Liver

Clinical

Summary

A 70-year-old man presents with vague abdominal pain and increasing jaundice. His past medical history is notable for chronic hepatitis C. A computed tomography scan reveals a left lobe mass in the liver. Gross examination of the hepatic lobectomy specimen reveals a cirrhotic liver with a tan soft 9.0 cm mass. Of note, the gallbladder and extrahepatic bile ducts are unremarkable on gross examination and histologically.

Master List 5604 Dysplastic nodule2484 Focal nodular hyperplasia2485 Hepatocellular carcinoma2277 Intrahepatic cholangiocarcinoma1518 Macroregenerative cirrhotic nodule

Criteria for

Diagnosis

and

Comments

The histological sections demonstrate a circumscribed hepatic parenchymal lesion comprised of sheets and trabeculae of polygonal neoplastic cells with eosinophilic granular cytoplasm. The nuclei of the tumor cells demonstrate marked atypia, ie, hyperchromasia, multinucleation, increased nucleus to cytoplasm ratio, and irregular nuclear membranes. There are clusters of neoplastic cells that demonstrate clear cytoplasm and others that form pseudoglands. Foci of vascular invasion are present. These features are those of hepatocellular carcinoma in a cirrhotic background.

Hepatocellular carcinoma (HCC) is the most common primary hepatic malignancy. It is the third leading cause of cancer-related deaths in the world; it is less common in the United States where it ranks ninth and is responsible for 12,000 deaths a year. It is more common in males and African Americans. China and Japan have the highest rates of HCC. The incidence is also high in developing countries and continues to rise in industrialized nations.

The majority of HCC arise in the background of chronic liver disease, with chronic viral hepatitis B and hepatitis C accounting for 80% of cases. Alcohol- and non-alcohol-related fatty liver disease, as well as tobacco use and increasing age, are also etiological factors. The ingestion of aflatoxin is linked to HCC in Asian and sub-Saharan African nations. Chronic liver injury due to hereditary factors like hemochromatosis and -1 antitrypsin deficiency increases the risk of HCC, while patients with inherited metabolic diseases, such as glycogen storage disease, also have higher risk of developing HCC. Lastly, HCC could develop from some variants of hepatic adenomas in the absence of chronic liver disease. The coexistence of multiple risk factors significantly increases the risk of cirrhosis which, independent of etiology,

44

Page 45: 2019 JPIP-A 病理診断生涯教育プログラムLeslie Dodd, MD FCAP Sabrina C. Sopha, MD FCAP Raul Gonzalez, MD FCAP Nichole L. Steidler, MD FCAP Christina Isacson, MD FCAP Justin

# 2019-05

Hepatocellular carcinoma

70 C CT9.0cm

(5604) Dysplastic nodule(2484) Focal nodular hyperplasia(2485) Hepatocellular carcinoma(2277) Intrahepatic cholangiocarcinoma(1518) Macroregenerative cirrhotic nodule

N/C

hepatocellular carcinoma HCC3 9 12,000

HCC

HCC B C 80

HCC

HCC HCCHCC

HCC

45

Page 46: 2019 JPIP-A 病理診断生涯教育プログラムLeslie Dodd, MD FCAP Sabrina C. Sopha, MD FCAP Raul Gonzalez, MD FCAP Nichole L. Steidler, MD FCAP Christina Isacson, MD FCAP Justin

is a major clinical risk factor for the development of HCC.

Grossly, HCC can be solitary or multiple. Oftentimes, a dominant nodule will be associated with smaller satellite lesions. A fibrous capsule separating the tumor from the surrounding stroma may be present.

Many grading systems exist for HCC, including the Edmondson and Steiner system, which divides the tumors into 4 grades from I to IV based on histological differentiation:

Grade I: Tumor cells with abundant cytoplasm and minimal nuclear atypiaGrade II: Mild nuclear atypia with prominent nucleoli, hyperchromasia, and nuclear

irregularityGrade III: Moderate nuclear atypia with greater hyperchromasia and nuclear irregularityGrade IV: Marked nuclear pleomorphism, marked hyperchromasia, and anaplastic giant

cells

Most HCC are either grade II or III. It should be noted that tumor grade is not a strong prognostic factor. Many experts instead prefer a simpler classification of well, moderately, and poorly differentiated tumor.

There is variation in both the architectural growth patterns and the degree of cytologic atypia. Neoplastic cells have abundant to moderate amounts of cytoplasm, which can be pink with Mallory hyaline and show fatty or clear cell change. Large round nuclei and prominent nucleoli are common. Solid, trabecular, and pseudoglandular growth patterns are frequently encountered. There is loss of the normal hepatic plate structure, as can be observed with the help of a disrupted reticulin stain. The presence of arteries unaccompanied by portal tracts is a common feature of HCCs.

HCCs are often easily recognized on routine H&E staining; however, histochemical and immunostains are helpful when the diagnosis is difficult. Diffuse expansion and/or loss of reticulin staining are frequent in hepatocellular carcinoma. CAM5.2 is positive in all HCCs.CD10 and polyclonal carcinoembryonic antigen (pCEA) are expressed by up to 90% of HCCs in a canalicular pattern. CD34 shows a strong diffuse sinusoidal staining pattern in HCCs. HepPar1 has shown increased sensitivity for well to moderately differentiated HCCs. Glypican 3 labels up to 85% of HCCs and does not stain a non-inflamed and noncirrhotic background liver. Overall, it is best to use a panel of markers when the diagnosis is in doubt, including arginase-1, one of the newer fairly specific and sensitive markers for HCC.

Some HCCs may arise in the absence of cirrhosis, one such example being fibrolamellarHCC. This subtype is rare and is encountered in younger patients than conventional HCC.Grossly and histologically, the presence of fibrous septae, similar but more diffuse than theseptae of focal nodular hyperplasia, is the hallmark of this neoplasm. Tumor cells grow intrabeculae between these septae. The neoplastic cells have very eosinophilic cytoplasm with frequent pale inclusions and cytoplasmic globules. These tumors can show CK7 and CK19labelling. They have traditionally been thought to have a better prognosis than conventional

46

Page 47: 2019 JPIP-A 病理診断生涯教育プログラムLeslie Dodd, MD FCAP Sabrina C. Sopha, MD FCAP Raul Gonzalez, MD FCAP Nichole L. Steidler, MD FCAP Christina Isacson, MD FCAP Justin

HCC

HCC I IV 4 EdmondsonSteiner

IIIIIIIV

HCC II III

HCC

HCC HEHCC /

CAM5.2 HCC CD10 pCEA90 HCC canalicular pattern CD34 HCC

HepPar1 HCCGlypican 3 85 HCC

HCC1

HCC HCCHCC

CK7 CK19HCC HCC

HCC

47

Page 48: 2019 JPIP-A 病理診断生涯教育プログラムLeslie Dodd, MD FCAP Sabrina C. Sopha, MD FCAP Raul Gonzalez, MD FCAP Nichole L. Steidler, MD FCAP Christina Isacson, MD FCAP Justin

HCC due to the absence of background cirrhosis, but recent evidence suggests that this tumor could be biologically similar or more aggressive than conventional HCC with early invasion into major hepatic vessels.

In the absence of screening programs, HCC is often diagnosed late in its course, and themedian survival following diagnosis is approximately 6-20 months. Surgical resection is themainstay of treatment; however, resection may not be possible due to the extent of underlying liver disease. For these patients, along with those meeting the Milan criteria (ie, a single tumor

than 3.0 cm, no evidence of gross vascular invasion, and no regional nodal or distant metastases), liver transplantation is an option. To decrease the size and/or number of tumors, patients are often treated with radiofrequency ablation, transarterial chemoembolization, cryoablation, systemic chemotherapy, or stereotactic radiotherapy.

For tumors confined to the liver, the 5-year survival rate is 28%. Tumors with regional spread (growth into nearby organs), the five-year survival rate is 7%. Once distant metastasesdevelop, survival time is as low as two years. Tumors that have been completely surgically removed and those that are diagnosed early may have a 5-year survival rate as high as 50%-70%.

Dysplastic nodules can be single or multiple. They are more commonly associated withcirrhotic livers but have been noted in chronic liver disease without significant fibrosis.Dysplastic nodules range from 0.5 - 5.0 cm in size and are associated with higher risk ofdevelopment of hepatocellular carcinoma. Grossly, dysplastic nodules differ from thebackground liver in their color, size, texture, and a degree of bulging on cut surface. Thehepatic plates can be up to 2 cells thick. Dysplasia within these lesions is classified as lowandhigh-grade. Low-grade dysplasia is characterized by mild increase in cellularity of amonotonous cell population without significant architectural atypia. Portal tracts are presentwithin the nodule. Large cell change, defined as cellular enlargement, nuclear pleomorphism and hyperchromasia, and multinucleation of hepatocytes, used to be regarded as a feature of low-grade dysplasia, but is now recognized as evidence of replicative senescence rather than a premalignant lesion. Low-grade dysplastic nodules are not easily distinguished from largemacroregenerative nodules. The presence of arteries unpaired with bile ducts and a clone-like cell population favors dysplasia. Highgrade dysplasia is characterized by small cell change, which has been described as a collection of hepatocytes with a decreased cell volume, minimal nuclear pleomorphism, an increased nucleus to cytoplasm ratio, and an increased nuclear density.

Macroregenerative cirrhotic nodules are collections of regenerative hepatocytes arising in abackground of cirrhosis. There can be multiple nodules in a cirrhotic liver. They are slightlylarger than the background liver nodules and vary in size from 1.0-5.0 cm. Hepatic plates are mildly thickened, 1 - 2 cells thick. Scattered entrapped portal tracts are present within the nodule. The hepatocytes within the nodule resemble those in the adjacent liver parenchyma. Mallory hyaline, foci of bile ductular reaction, and inflammation can be present within a

48

Page 49: 2019 JPIP-A 病理診断生涯教育プログラムLeslie Dodd, MD FCAP Sabrina C. Sopha, MD FCAP Raul Gonzalez, MD FCAP Nichole L. Steidler, MD FCAP Christina Isacson, MD FCAP Justin

HCC6 20

5.0cm 3 3.0cm

5 285 7 2

5 50 70

dysplastic nodule0.5

5.0cm2

N/C

macroregenerative cirrhotic nodule

1.0 5.0cm 1 2

49

Page 50: 2019 JPIP-A 病理診断生涯教育プログラムLeslie Dodd, MD FCAP Sabrina C. Sopha, MD FCAP Raul Gonzalez, MD FCAP Nichole L. Steidler, MD FCAP Christina Isacson, MD FCAP Justin

macroregenerative nodule.

Intrahepatic cholangiocarcinoma (ICC) is an intrahepatic malignancy with biliary epithelialdifferentiation. It comprises 10% of primary liver malignancies. Any portion of the intrahepatic biliary network can give rise to this tumor. The age-specific incidence is highest in patients over 85 years. It is more commonly encountered in men and in Southeast Asian countries. Biliary and hepatic diseases such as primary sclerosing cholangitis, hepatolithiasis, parasitic infection of the biliary tract (liver flukes Clonorchis sinensis or Opisthorchis viverrini), biliary malformations, and non-biliary causes of cirrhosis (eg, viral hepatitis) are all risk factors for ICC. In addition, exposure to Thorotrast has also been associated with the development of ICCs.

ICCs can arise in association with biliary intraepithelial neoplasia (BilIN). The majority of ICCs are adenocarcinomas. The cells are columnar/cuboidal and form variably sized tubular structures that resemble bile ducts forming acinar, cord-like or papillary patterns of growth. The nuclei are round and centrally located. The glands are surrounded by acellular fibrous stroma with variable amounts of inflammation. Adenosquamous, squamous, clear cell, signet ring cell, lymphoepithelioma-like, mucinous, and sarcomatous histologic variants have been described. Mucin stains (mucicarmine, diastase-periodic acid Schiff, alcian blue) are positive in the tumor. CK7 and CK19 are often expressed by the neoplastic cells as are carcinoembryonic antigen and epithelial membrane antigen. ICC is an aggressive cancer that carries a poor prognosis and typically spreads by direct invasion, along hepatic plates and nerves.

Focal nodular hyperplasia is a benign condition that develops as a result of shunting of arterial blood flow which leads to the production of fibrous bands. It is a benign and nonneoplasticcondition. Histologically, the lesion is comprised of nodules of benign hepatocytes with intervening thin fibrous septae. The reticulin staining pattern demonstrates normal-thickness (1 - 2 cells) hepatocyte trabeculae. Ductular reaction can be seen in the fibrous septae. The background liver is non-cirrhotic. Glutamine synthetase demonstrates map-like staining which can help differentiate from inflammatory hepatic adenomas.

Educational

Questions

1. Which of the following is correct regarding hepatocellular carcinoma (HCC)?6064 Clonorchis sinensis and Opisthorchis viverrini infection are important risk factors.6065 HCCs are more frequently encountered in males and in African Americans.6066 Portal tracts are readily identified within the tumor.6067 The lesion develops as result of shunting of arterial blood flow, which leads to the

production of fibrous bands around nodules of hepatocytes.6068 Tumor grade is a strong prognostic factor.

50

Page 51: 2019 JPIP-A 病理診断生涯教育プログラムLeslie Dodd, MD FCAP Sabrina C. Sopha, MD FCAP Raul Gonzalez, MD FCAP Nichole L. Steidler, MD FCAP Christina Isacson, MD FCAP Justin

intrahepatic cholangiocarcinoma ICC10

85Clonorchis sinensis

Opisthorchis viverriniICC ICC

ICC biliary intraepithelial neoplasia BilIN ICC

PASCEA EMA CK7 CK19 ICC

focal nodular hyperplasia

1 2

1 31. HCC

(6064) Clonorchis sinensis Opisthorchis viverrini

(6065) HCC(6066)(6067)

(6068)

51

Page 52: 2019 JPIP-A 病理診断生涯教育プログラムLeslie Dodd, MD FCAP Sabrina C. Sopha, MD FCAP Raul Gonzalez, MD FCAP Nichole L. Steidler, MD FCAP Christina Isacson, MD FCAP Justin

2. Which of the following is correct of the fibrolamellar variant of hepatocellular carcinoma?6069 Neoplastic cells with eosinophilic cytoplasm with frequent pale inclusions and

cytoplasmic globules grow between dense fibrous septae.6070 Small cell change is a prominent feature.6071 The tumor is distinguished by CK7- and CK19-positive glands of cuboidal cells

embedded in fibrotic stroma.6072 The tumor is comprised of regenerative hepatocytes arising in a background of

cirrhosis with no expansion of the hepatic plates.6073 The tumor is the result of shunting of arterial blood flow, which leads to the

production of fibrous bands.

3. Which of the following are histologic features of a dysplastic hepatic nodule?6074 Collections of hepatocytes with a decreased cell volume, minimal nuclear

pleomorphism, an increased nucleus to cytoplasm ratio, and increased nuclear density

6075 Entrapped portal tracts with hepatocytes sharing the same histologic features as those in adjacent parenchyma

6076 Expanded and disrupted hepatic plate architecture with neoplastic cells growing in solid, trabecular, and pseudoglandular patterns

6077 Marked nuclear pleomorphism, marked hyperchromasia, and anaplastic giant cells6078 Nodular expansion of hepatocytes with intervening thin fibrous septae

References 1. Edmondson HA, Steiner PE. Primary carcinoma of the liver: a study of 100 cases among 48,900 necropsies. Cancer. 1954;7(3):462-503.

2. Paradis V. Histopathology of Hepatocellular Carcinoma. Recent Results Cancer Res.2013;190:21-32.

3. Schlageter M, Terracciano ML, D’Angelo S, Sorrentin P. Histopathology of hepatocellular carcinoma. World J Gastroenterol. 2014; 20(43):15955---15964.

Safia N. Salaria, MDSurgical Pathology Committee

Vanderbilt University Medical CenterNashville, TN

52

Page 53: 2019 JPIP-A 病理診断生涯教育プログラムLeslie Dodd, MD FCAP Sabrina C. Sopha, MD FCAP Raul Gonzalez, MD FCAP Nichole L. Steidler, MD FCAP Christina Isacson, MD FCAP Justin

2.(6069)

(6070)(6071) CK7 CK19

(6072)(6073)

3. (6074) N/C

(6075)(6076)

(6077)(6078)

97

1. Edmondson HA, Steiner PE. Primary carcinoma of the liver: a study of 100 cases among 48,900 necropsies. Cancer. 1954;7(3):462-503.

2. Paradis V. Histopathology of Hepatocellular Carcinoma. Recent Results Cancer Res.2013;190:21-32.

3. Schlageter M, Terracciano ML, D’Angelo S, Sorrentin P. Histopathology of hepatocellular carcinoma. World J Gastroenterol. 2014; 20(43):15955---15964.

Safia N. Salaria, MDSurgical Pathology Committee

Vanderbilt University Medical CenterNashville, TN

53

Page 54: 2019 JPIP-A 病理診断生涯教育プログラムLeslie Dodd, MD FCAP Sabrina C. Sopha, MD FCAP Raul Gonzalez, MD FCAP Nichole L. Steidler, MD FCAP Christina Isacson, MD FCAP Justin

Case # 2019-06

Diagnosis Poorly differentiated squamous cell carcinoma

Site Lung

Clinical

Summary

A 70-year-old woman with a long history of smoking presents with weight loss, cough, andshortness of breath of several months’ duration. A chest radiograph shows a large denseopacity in the right upper lobe with traction of the minor fissure and hilar adenopathy.Computerized tomography (CT) scan reveals a 12.0 cm mass with enlarged hilar lymphnodes. Mediastinoscopy is negative. Right upper lobectomy along with mediastinal lymphnode dissection is performed. Gross examination reveals a 12.5 cm yellow-white mass withcentral cavitation that seems to be invading into the visceral pleura, causing retraction. Thetumor is positive for cytokeratin 5/6 and p63 and negative for TTF-1, Napsin A, HMB-45,SOX-10, cytokeratin 7, and cytokeratin 20.

Master List 2658 Large cell carcinoma1165 Melanoma1421 Pleomorphic carcinoma1388 Poorly differentiated adenocarcinoma5605 Poorly differentiated squamous cell carcinoma

Criteria for

Diagnosis

and

Comments

Histologic sections show sheets or nests of large polygonal cells with vesicular nuclei, prominent nucleoli, and a moderate amount of cytoplasm. On most slides, the tumor shows characteristics of a non-small cell lung carcinoma (NSCLC); however, some slides show focal squamous differentiation. In these cases, it is important to perform immunostains to confirm squamous differentiation and exclude adenocarcinoma or adenosquamous carcinoma, as those diagnoses would warrant further molecular testing. The tumor cells express CK5/6 and p63 (markers for squamous cell carcinoma) and negative for TTF-1 and Napsin A (markers for lung adenocarcinoma), confirming the diagnosis of poorly differentiated squamous cell carcinoma.

Squamous cell carcinoma (SCC) is a malignant epithelial tumor showing keratinization and/or intercellular bridges that arises from bronchial epithelium. More than 90% of squamous cell lung carcinomas occur in cigarette smokers. The majority of squamous cell lung carcinomas arise centrally in the mainstem, lobar, or segmental bronchi. The primary tumor and its central extent of disease is usually best demonstrated by CT scan. The cytology of tumor cells depends on the histologic differentiation and type of sampling. On cytologic smears, the tumor cells may occur singly or in clusters and may show bizarre shapes such as spindle-shaped and tadpole-shaped cells. Cohesive aggregates may occur in flat sheets with elongated or spindled cells. On histologic examination, squamous cell carcinoma shows keratinization, pearl formation, and/or intercellular bridges. These features vary with degree of differentiation, being prominent in well-differentiated tumors and focal in poorly differentiated tumors.

54

Page 55: 2019 JPIP-A 病理診断生涯教育プログラムLeslie Dodd, MD FCAP Sabrina C. Sopha, MD FCAP Raul Gonzalez, MD FCAP Nichole L. Steidler, MD FCAP Christina Isacson, MD FCAP Justin

# 2019-06

Poorly differentiated squamous cell carcinoma

70 XCT

12.0cm12.5cm

5/6 p63TTF-1 Napsin A HMB-45 SOX-10 7 20

(2658) Large cell carcinoma(1165) Melanoma(1421) Pleomorphic carcinoma(1388) Poorly differentiated adenocarcinoma(5605) Poorly differentiated squamous cell carcinoma

non-small cell lung carcinoma NSCLC

CK5/6 p63TTF-1 Napsin A

squamous cell carcinoma SCC90

CT

55

Page 56: 2019 JPIP-A 病理診断生涯教育プログラムLeslie Dodd, MD FCAP Sabrina C. Sopha, MD FCAP Raul Gonzalez, MD FCAP Nichole L. Steidler, MD FCAP Christina Isacson, MD FCAP Justin

On gross examination, SCCs are usually white or grey and, depending on the severity of fibrosis, firm with focal carbon pigment deposits in the center and retractions at the periphery. The tumor may grow to a large size and may cavitate. Central tumors form intraluminal polypoid masses and/or infiltrate through the bronchial wall into the surrounding lung parenchyma, occluding the bronchial lumen and resulting in stasis of bronchial secretions, atelectasis, bronchial dilatation, obstructive lipoid pneumonia, and bronchopneumonia. A minority of cases may arise in small peripheral airways.

Detection of aneuploidy by DNA measurement has been shown to be a poor prognostic factor for SCC of the lung. ErbB (EGFR, HER2, KRAS) pathway abnormalities are common in non-small cell carcinoma but absent in SCC. Stage for stage, the survival rate for SCC is significantly better than for adenocarcinoma. Approximately 80% of patients with resected stage 1 (T1 N0 M0) SCC are alive at 5 years after diagnosis compared to approximately 70% of similarly staged adenocarcinomas.

Large cell carcinoma (not to be confused with large cell neuroendocrine carcinoma) is an undifferentiated non-small cell carcinoma that lacks the cytologic and architectural features of small cell carcinoma and glandular or squamous differentiation. Large cell carcinoma accounts for approximately 9% of all lung cancers. It is a diagnosis of exclusion made after ruling out the presence of a component of SCC, adenocarcinoma, or small cell carcinoma. Large cell carcinomas share the molecular and genetic alterations commonly seen in NSCLC, since it is a poorly differentiated tumor arising from the same stem cells, exposed to the same carcinogens. KRAS mutations, TP53 mutations and Rb pathway alteration (loss of p16INK4, hyperexpression of cyclin D1 or cyclin E) occurs with the same frequency as in other NSCLC.

Melanoma metastasizing to the lung is in the differential diagnosis of these tumors, especially when cytokeratin immunostaining is negative. In these situations, performing HMB-45, SOX-10, and Melan-A is helpful for diagnosis.

Pleomorphic carcinoma is a rare, poorly differentiated subtype of lung carcinoma defined by the 2015 World Health Organization as a squamous cell carcinoma, adenocarcinoma, or undifferentiated NSCLC that contains at least 10% spindle and/or giant cells, or a carcinoma consisting only of spindle and giant cells. While the presented case is poorly differentiated, no significant spindle or giant cell component is noted; therefore, it does not meet criteria for diagnosis of pleomorphic carcinoma.

Poorly differentiated adenocarcinoma or the solid variant of lung adenocarcinoma is composed of sheets of polygonal cells lacking acini, tubules, and papillae, but with mucin present in at least 5 tumor cells in each of 2 high-power fields, confirmed with histochemical stains for mucin. Squamous carcinomas and large cell carcinomas of the lung may show rare cells with intracellular mucin, but this does not indicate classification as adenocarcinoma. These tumors express cytokeratin 7, TTF-1 and Napsin A. TTF-1 is especially positive in the better differentiated tumors. Common genetic alterations include EGFR, KRAS and BRAF mutations; translocation of ALK, ROS1, and RET; and amplification of MET and FGFR1.

56

Page 57: 2019 JPIP-A 病理診断生涯教育プログラムLeslie Dodd, MD FCAP Sabrina C. Sopha, MD FCAP Raul Gonzalez, MD FCAP Nichole L. Steidler, MD FCAP Christina Isacson, MD FCAP Justin

DNAErbB EGFR HER2 KRAS

1 T1 N0 M0 5 80 70

large cell carcinoma large cell neuroendocrine carcinoma

9%

KRAS TP53 Rb p16INK4 cyclin D1 cyclin E

HMB-45 SOX-10 Melan-A

pleomorphic carcinoma 2015 WHO10

2 5

7TTF-1 Napsin A TTF-1

EGFR KRAS BRAF ALK ROS1 RET MET FGFR1

57

Page 58: 2019 JPIP-A 病理診断生涯教育プログラムLeslie Dodd, MD FCAP Sabrina C. Sopha, MD FCAP Raul Gonzalez, MD FCAP Nichole L. Steidler, MD FCAP Christina Isacson, MD FCAP Justin

Educational

Questions

1. Which of the following is most distinctive of poorly differentiated squamous cell carcinoma (SCC)?

6079 Immunoreactivity for p63 and p406080 Immunoreactivity for HMB-452205 Presence of necrosis5942 Translocations involving chromosome 126081 Tumor cells forming glands

2. Which of the following is true of poorly differentiated lung SCC?6082 Distant metastases are not seen.6083 Prognosis is independent of stage.6084 Stage for stage, survival rate is significantly better than for adenocarcinoma.6085 These tumors show large areas of keratinization.6086 Tumor cells express TTF-1.

3. Which of the following is most typical of lung adenocarcinoma?6087 Common alterations include EGFR, KRAS, and BRAF6088 Express p63 and p406089 Grossly well-circumscribed6090 Least commonly seen in the periphery of the lung6091 Most are central tumors

References 1. Lindeman NI, Cagle PT, Beasley MB, et al. Molecular testing guideline for selection of lung cancer patients for EGFR and ALK tyrosine kinase inhibitors: guideline from the College of American Pathologists, International Association for the Study of Lung Cancer, and Association for Molecular Pathology. J Thorac Oncol. 2013;8(7):823-859.

2. Rekhtman N, Paik PK, Arcila ME, et al. Clarifying the spectrum of driver oncogene mutations in biomarker-verified squamous carcinoma of lung: lack of EGFR/KRAS and presence of PIK3CA/AKT1 mutations. Clin Cancer Res. 2012;18:1167-1176.

3. Travis WD, Brambilla E, Burke AP, Marx A, Nicholson AG. WHO Classification of Tumours of the Lung, Pleura, Thymus and Heart. International Agency for Research on Cancer, Lyon, FR: International Agency for Research on Cancer; 2015.

Kirtee Raparia, MDSurgical Pathology Committee

Kaiser Permanente Medical CenterSanta Clara, CA

58

Page 59: 2019 JPIP-A 病理診断生涯教育プログラムLeslie Dodd, MD FCAP Sabrina C. Sopha, MD FCAP Raul Gonzalez, MD FCAP Nichole L. Steidler, MD FCAP Christina Isacson, MD FCAP Justin

1 31.

(6079) p63 p40(6080) HMB-45(2205)(5942) 12(6081)

2. (6082)(6083)(6084)(6085)(6086) TTF-1

3. (6087) EGFR KRAS BRAF(6088) p63 p40(6089)(6090)(6091)

97

1. Lindeman NI, Cagle PT, Beasley MB, et al. Molecular testing guideline for selection of lung cancer patients for EGFR and ALK tyrosine kinase inhibitors: guideline from the College of American Pathologists, International Association for the Study of Lung Cancer, and Association for Molecular Pathology. J Thorac Oncol. 2013;8(7):823-859.

2. Rekhtman N, Paik PK, Arcila ME, et al. Clarifying the spectrum of driver oncogene mutations in biomarker-verified squamous carcinoma of lung: lack of EGFR/KRAS and presence of PIK3CA/AKT1 mutations. Clin Cancer Res. 2012;18:1167-1176.

3. Travis WD, Brambilla E, Burke AP, Marx A, Nicholson AG. WHO Classification of Tumours of the Lung, Pleura, Thymus and Heart. International Agency for Research on Cancer, Lyon, FR: International Agency for Research on Cancer; 2015.

Kirtee Raparia, MDSurgical Pathology Committee

Kaiser Permanente Medical CenterSanta Clara, CA

59

Page 60: 2019 JPIP-A 病理診断生涯教育プログラムLeslie Dodd, MD FCAP Sabrina C. Sopha, MD FCAP Raul Gonzalez, MD FCAP Nichole L. Steidler, MD FCAP Christina Isacson, MD FCAP Justin

Case # 2019-07

Diagnosis Seminoma

Site Testis

Clinical

Summary

A 40-year-old man presents with a painless, palpable testicular mass. Scrotal ultrasonography shows a 3.5 cm homogeneous, hypoechoic intratesticular mass. Serum tumor markers show normal values for alpha fetoprotein (AFP), human chorionic gonadotropin (hCG), and lactate dehydrogenase (LDH). An orchiectomy is performed which reveals a 3.5 cm solid, soft, circumscribed, homogeneous, tan mass confined to the testicular parenchyma.

Master List 2716 Diffuse large B-cell lymphoma, NOS1994 Embryonal carcinoma5607 Monophasic choriocarcinoma2383 Seminoma2341 Sertoli cell tumor5608 Spermatocytic tumor2467 Yolk sac tumor

Criteria for

Diagnosis

and

Comments

Histologic sections reveal a neoplasm composed of nests and sheets of large round cells with abundant clear cytoplasm. Intervening fibrovascular septae are present which contain variable numbers of lymphocytes. The cells are uniform and non-overlapping and nuclei have finely granular chromatin with prominent nucleoli. Intratubular growth may be seen in some sections and adjacent seminiferous tubules show germ cell neoplasia in situ (GCNIS, formerly known as intratubular germ cell neoplasia [ITGCN]) in some sections. Based on the serologic, gross, and histologic features, this is most consistent with a seminoma.

Seminoma is a malignant germ cell tumor that arises from primordial germ cells. It is the most common pure germ cell tumor and accounts for approximately 50% of all germ cell tumors. A tumor with identical morphology located in the ovary is referred to as dysgerminoma and germinoma in other parts of the body. The usual age range is 30-49 years; it is rare in prepubertal children and men older than 70 years. Risk factors include family history, previous germ cell tumor, subfertility and cryptorchidism. Most patients present with a unilateral painless testicular mass or swelling, although other symptoms may include localized testicular pain, symptoms due to metastasis (lower back pain in the case of involvement of retroperitoneal lymph nodes), or rarely, paraneoplastic syndromes (gynecomastia, exophthalmos, hypercalcemia, polycythemia, hemolytic anemia, and limbic encephalopathy). Human chorionic gonadotropin (hCG) levels are usually not increased but may be increased when there is a component of syncytiotrophoblastic cells in the tumor. Lactate dehydrogenase (LDH) may be increased, especially in advanced-stage disease. Alpha-fetoprotein (AFP) is

60

Page 61: 2019 JPIP-A 病理診断生涯教育プログラムLeslie Dodd, MD FCAP Sabrina C. Sopha, MD FCAP Raul Gonzalez, MD FCAP Nichole L. Steidler, MD FCAP Christina Isacson, MD FCAP Justin

# 2019-07

Seminoma

40 3.5cmAFP hCG LDH

3.5cm

(2716) B NOS Diffuse large B-cell lymphoma, NOS(1994) Embryonal carcinoma(5607) Monophasic choriocarcinoma(2383) Seminoma(2341) Sertoli cell tumor(5608) Spermatocytic tumor(2467) Yolk sac tumor

germ cell neoplasia in situGCNIS intratubular germ cell neoplasia [ITGCN]

seminoma primordial germ cell 50

dysgerminomagerminoma 30 49 70

human chorionic gonadotropin hCG

lactate dehydrogenase LDH- alpha-fetoprotein AFP

AFP

61

Page 62: 2019 JPIP-A 病理診断生涯教育プログラムLeslie Dodd, MD FCAP Sabrina C. Sopha, MD FCAP Raul Gonzalez, MD FCAP Nichole L. Steidler, MD FCAP Christina Isacson, MD FCAP Justin

typically not increased or minimally increased in cases of liver metastasis; if AFP levels aresignificantly increased, the tumor is typically clinically treated as a nonseminomatous germ cell tumor regardless of the tumor morphology.

Seminoma usually has a solid, homogeneous, lobular, tan to gray-white gross appearance. Small foci of necrosis and/or punctate hemorrhage may be seen but are not prominent. They are usually composed of large, polygonal cells with abundant clear cytoplasm. Cells are evenly spaced with distinct cytoplasmic membranes. Nuclei are regular and round and show finely granular chromatin with prominent nucleoli. The cells may be arranged in solid sheets, nests, interstitial, tubular, or trabecular patterns with intervening fibrovascular septae containing numerous lymphocytes and plasma cells. Granulomatous inflammation is evident in approximately 30% of cases and may be extensive, obscuring tumor cells. Foci of fibrosis and sclerosis may be seen in areas of tumor regression. Hemorrhage and necrosis are rarely present, and if prominent should warrant careful search for other nonseminomatous germ cell tumor components. Syncytiotrophoblastic cells may be present, usually within areas of hemorrhage. GCNIS is frequently seen in the surrounding seminiferous tubules.

Seminoma is usually positive for PLAP, OCT 3/4, NANOG, CD117, Podoplanin (D2-40),SALL4 and vimentin immunohistochemical stains and negative for cytokeratin, alphafetoprotein (AFP), Beta-hCG, CD30, glypican 3, and inhibin immunohistochemicalstains. PAS without diastase highlights abundant glycogen particles within the cytoplasm.

The prognosis of seminoma is excellent relative to other malignant germ cell tumors.Prognosis worsens with increased pathologic stage, increased tumor size, involvement of rete testis and/or epididymis, and presence of intertubular growth in > 3 high-power fields. Lymphovascular invasion is not an independent prognostic factor, and increased mitoses (> 3 mitoses per high power field), as previously used to define so-called “anaplastic seminoma,” has not been proven to be an adverse prognostic factor. Stage I tumors are typically treated with radical orchiectomy followed by either surveillance with measurement of serum markers, chest x-ray and computed tomography, single agent chemotherapy, or radiation therapy. Stage II tumors are treated with radical orchiectomy and radiation therapy or combination chemotherapy and stage III tumors are treated with radical orchiectomy and multidrug chemotherapy. For low-stage tumors, risk of recurrence increases with primary tumor size > 4.0 cm, rete testis or epididymal involvement, and possibly vascular invasion. For high stage tumors, age greater than or equal to 40 years and non-pulmonary visceral metastases are adverse prognostic factors.

The differential diagnosis of seminoma includes other germ cell tumors, such as embryonal carcinoma, yolk sac tumor, monophasic choriocarcinoma, and spermatocytic tumor, other testicular tumors such as Sertoli cell tumor and systemic tumors such as lymphoma. Adequate evaluation for other germ cell tumor components must be performed to rule out a mixed germ cell tumor or nonseminomatous germ cell tumor, as these have a worse prognosis than pure seminoma.

62

Page 63: 2019 JPIP-A 病理診断生涯教育プログラムLeslie Dodd, MD FCAP Sabrina C. Sopha, MD FCAP Raul Gonzalez, MD FCAP Nichole L. Steidler, MD FCAP Christina Isacson, MD FCAP Justin

30

GCNIS

PLAP OCT 3/4 NANOG CD117 D2-40 SALL4AFP -hCG CD30 glypican 3

PAS

HPF 4

anaplastic seminoma 3HPF I

X CTII

III4.0 cm

40

embryonal carcinoma yolk sac tumormonophasic choriocarcinoma spermatocytic tumor

Sertoli cell tumor mixed germ cell tumor

63

Page 64: 2019 JPIP-A 病理診断生涯教育プログラムLeslie Dodd, MD FCAP Sabrina C. Sopha, MD FCAP Raul Gonzalez, MD FCAP Nichole L. Steidler, MD FCAP Christina Isacson, MD FCAP Justin

Embryonal carcinoma may resemble seminoma in that both tumors may show a solidpattern of cells displaying prominent nucleoli. The presence of other architectural patterns (glandular or papillary) is seen in embryonal carcinoma, as is marked nuclear pleomorphism, nuclear overlapping, and indistinct cell borders. Seminoma has more well-defined cytoplasmic membranes, less crowded and less pleomorphic nuclei, fibrous septa, and a prominent lymphocytic infiltrate. Immunohistochemical positivity for CD30 and cytokeratin and negativity for CD117 and D2-40 support the diagnosis of embryonal carcinoma.

Yolk sac tumor may also resemble seminoma in that both may show a solid pattern of cells with abundant clear cytoplasm and distinct cytoplasmic membranes. Yolk sac tumor frequently shows other more characteristic architectural patterns (microcystic and reticular), has hyaline globules, may have Schiller-Duval bodies, and lacks fibrous bands and the lymphocytic infiltrate typically seen in seminoma. Positivity for glypican 3, cytokeratin and AFP and negativity for NANOG, OCT 3/4, and D2-40 support the diagnosis of yolk sac tumor.

Monophasic choriocarcinoma may enter the differential diagnosis of a seminoma. This is a rare tumor characterized by mononucleated cells of variable sizes with more frequent hemorrhage and necrosis than seminoma. Both tumors may show syncytiotrophoblastic cells. Choriocarcinomas are positive for Beta-hCG and negative for CD117 and D2-40, unlike seminomas.

Spermatocytic tumor may resemble seminoma, however usually occurs in an older age group and has no association with GCNIS. There are 3 distinct tumor cell types (small, intermediate-sized, and giant cells) and there is no lymphocytic infiltration, granulomatous inflammation, or fibrous septa. This tumor is positive for CD117 and SALL4 and negative for cytokeratin, PLAP, NANOG, OCT 3/4, D2-40, AFP, CD30, and Glypican 3.

Sertoli cell tumor may resemble seminoma; however, Sertoli cell tumors usually showtubular or cystic growth and have smaller, less pleomorphic nuclei and lower mitotic rates than seminoma. Sertoli cell tumor also lacks fibrous septae, lymphocytic inflammation, and granulomatous inflammation. The finding of GCNIS is a helpful feature in the diagnosis of seminoma. Sertoli cell tumors are positive for markers of sex cord-stromal differentiation (inhibin) and cytokeratin and negative for germ cell markers (SALL4 and PLAP), D2-40, and OCT 3/4.

Lymphoma may resemble seminoma. The most common type of lymphoma involving the testis is diffuse large B-cell lymphoma and this usually affects an older age group than seminoma. Lymphoma shows more interstitial growth between the tubules and lacks GCNIS. Immunohistochemical stains for lymphoid markers (CD45 and T-cell or Bcell lineage specific markers) and negativity for germ cell markers aid in the diagnosis of lymphoma.

64

Page 65: 2019 JPIP-A 病理診断生涯教育プログラムLeslie Dodd, MD FCAP Sabrina C. Sopha, MD FCAP Raul Gonzalez, MD FCAP Nichole L. Steidler, MD FCAP Christina Isacson, MD FCAP Justin

embryonal carcinoma glandular papillary

CD30 CD117 D2-40

yolk sac tumor

microcystic reticular hyaline globule Schiller-DuvalGlypican 3

AFP NANOG OCT 3/4 D2-40

monophasic choriocarcinoma

-hCG CD117D2-40

spermatocytic tumor GCNIS 3 small intermediate

giant cell CD117SALL4 PLAP NANOG OCT 3/4 D2-40 AFP CD30 Glypican 3

Sertoli cell tumor

GCNIS

SALL4 PLAP D2-40 OCT 3/4

BGCNIS CD45 T

B

65

Page 66: 2019 JPIP-A 病理診断生涯教育プログラムLeslie Dodd, MD FCAP Sabrina C. Sopha, MD FCAP Raul Gonzalez, MD FCAP Nichole L. Steidler, MD FCAP Christina Isacson, MD FCAP Justin

Immunohistochemical Staining Pattern in Germ Cell TumorsSeminoma Embryonal

carcinomaYolk sac tumor

Choriocarcinoma Spermatocytic tumor

AE1/3 -/+ + + + -AFP - - + - -CD30 - + - - -Glypican 3 - - + + -CD117 + - +/- - +NANOG + + - - -OCT 3/4 + + - - -PLAP + + + + -D2-40 + - - - -SALL4 + + + + +Beta-hCG - - - + -

Educational

Questions

1. Which of the following tumors may show extensive granulomatous inflammation?6092 Monophasic choriocarcinoma1966 Seminoma1154 Sertoli cell tumor6093 Spermatocytic tumor2472 Yolk sac tumor

2. Which of the following immunohistochemical findings is most characteristic of seminoma?6094 Glypican 3 (+), OCT 3/4 (-), CD117 (+), D2-40 (-), CD30 (-)6095 Glypican 3 (+), OCT 3/4 (-), CD117 (-), D2-40 (-), CD30 (-)6096 Glypican 3 (-), OCT 3/4 (+), CD117 (+), D2-40 (+), CD30 (-)6097 Glypican 3 (-), OCT 3/4 (-), CD117 (+), D2-40 (-), CD30 (-)6098 Glypican 3 (-), OCT 3/4 (+), CD117 (-), D2-40 (-), CD30 (+)

3. Which of the following is true regarding the prognosis of seminoma?6099 Elevated AFP is an adverse prognostic factor.6100 Increased mitoses (> 3 mitoses per high power field) is an adverse prognostic factor.6101 Intertubular growth (> 3 high power fields) is an adverse prognostic factor.6102 Lymphovascular invasion is an adverse prognostic factor.6103 Patient age < 40 is an adverse prognostic factor.

66

Page 67: 2019 JPIP-A 病理診断生涯教育プログラムLeslie Dodd, MD FCAP Sabrina C. Sopha, MD FCAP Raul Gonzalez, MD FCAP Nichole L. Steidler, MD FCAP Christina Isacson, MD FCAP Justin

Seminoma Embryonal carcinoma

Yolk sac tumor

Choriocarcinoma Spermatocytic tumor

AE1/3 -/+ + + + -AFP - - + - -CD30 - + - - -Glypican 3 - - + + -CD117 + - +/- - +NANOG + + - - -OCT 3/4 + + - - -PLAP + + + + -D2-40 + - - - -SALL4 + + + + +Beta-hCG - - - + -

1 31.

(6092) Monophasic choriocarcinoma(1966) Seminoma(1154) Sertoli cell tumor(6093) Spermatocytic tumor(2472) Yolk sac tumor

2.(6094) Glypican 3 (+), OCT 3/4 (-), CD117 (+), D2-40 (-), CD30 (-)(6095) Glypican 3 (+), OCT 3/4 (-), CD117 (-), D2-40 (-), CD30 (-)(6096) Glypican 3 (-), OCT 3/4 (+), CD117 (+), D2-40 (+), CD30 (-)(6097) Glypican 3 (-), OCT 3/4 (-), CD117 (+), D2-40 (-), CD30 (-)(6098) Glypican 3 (-), OCT 3/4 (+), CD117 (-), D2-40 (-), CD30 (+)

3. (6099) AFP(6100) 3 HPF(6101) 4 HPF(6102)(6103) 40

97

67

Page 68: 2019 JPIP-A 病理診断生涯教育プログラムLeslie Dodd, MD FCAP Sabrina C. Sopha, MD FCAP Raul Gonzalez, MD FCAP Nichole L. Steidler, MD FCAP Christina Isacson, MD FCAP Justin

References 1. Tickoo SK, Hutchinson B, Bacik J, et al. Testicular seminoma: a clinicopathologic and immunohistochemical study of 105 cases with special reference to seminomas with atypical features. Int J Surg Pathol. 2002;10(1):23-32.

2. Ulbright TM, Amin MB, Balzer B, et al. Germ cell tumours. In Moch H, Humphrey PA, Ulbright TM, Reuter VE, eds. WHO Classification of Tumours of the Urinary System and Male Genital Organs. 4th ed. Lyon, FR: IARC press;2016:203-205.

3. Ulbright TM, Young RH. Seminoma with tubular, microcystic, and related patterns: a study of 28 cases of unusual morphologic variants that often cause confusion with yolk sac tumor. Am J Surg Pathol. 2005;29(4):500-505.

4. Ulbright TM. Germ cell tumors of the gonads: a selective review emphasizing problems in differential diagnosis, newly appreciated, and controversial issues. Mod Pathol. 2005;Suppl 2:S61-79.

5. Young RH. Testicular tumors – some new and a few perennial problems. Arch Pathol Lab Med. 2008;132(4):548-564.

6. Zengerling F, Kunath F, Jensen K, Ruf C, Schmidt S, Spek A. Prognostic factors for tumor recurrence in patients with clinical stage I seminoma undergoing surveillance – a systematic review. Urol Oncol. 2018; 36(10):448-458.

Danielle E. Westfall, MDSurgical Pathology Committee

TOPA DiagnosticsThousand Oaks, CA

68

Page 69: 2019 JPIP-A 病理診断生涯教育プログラムLeslie Dodd, MD FCAP Sabrina C. Sopha, MD FCAP Raul Gonzalez, MD FCAP Nichole L. Steidler, MD FCAP Christina Isacson, MD FCAP Justin

1. Tickoo SK, Hutchinson B, Bacik J, et al. Testicular seminoma: a clinicopathologic and immunohistochemical study of 105 cases with special reference to seminomas with atypical features. Int J Surg Pathol. 2002;10(1):23-32.

2. Ulbright TM, Amin MB, Balzer B, et al. Germ cell tumours. In Moch H, Humphrey PA, Ulbright TM, Reuter VE, eds. WHO Classification of Tumours of the Urinary System and Male Genital Organs. 4th ed. Lyon, FR: IARC press;2016:203-205.

3. Ulbright TM, Young RH. Seminoma with tubular, microcystic, and related patterns: a study of 28 cases of unusual morphologic variants that often cause confusion with yolk sac tumor. Am J Surg Pathol. 2005;29(4):500-505.

4. Ulbright TM. Germ cell tumors of the gonads: a selective review emphasizing problems in differential diagnosis, newly appreciated, and controversial issues. Mod Pathol. 2005;Suppl 2:S61-79.

5. Young RH. Testicular tumors – some new and a few perennial problems. Arch Pathol Lab Med. 2008;132(4):548-564.

6. Zengerling F, Kunath F, Jensen K, Ruf C, Schmidt S, Spek A. Prognostic factors for tumor recurrence in patients with clinical stage I seminoma undergoing surveillance – a systematic review. Urol Oncol. 2018; 36(10):448-458.

Danielle E. Westfall, MDSurgical Pathology Committee

TOPA DiagnosticsThousand Oaks, CA

69

Page 70: 2019 JPIP-A 病理診断生涯教育プログラムLeslie Dodd, MD FCAP Sabrina C. Sopha, MD FCAP Raul Gonzalez, MD FCAP Nichole L. Steidler, MD FCAP Christina Isacson, MD FCAP Justin

Case # 2019-08

Diagnosis Well-differentiated liposarcoma

Site Retroperitoneum

Clinical

Summary

A 70-year-old woman presents with increasing abdominal girth, discomfort, and bilateral legswelling over several months. She also reports a weight gain of 60 lbs during this period, earlysatiety, and decreased appetite. Radiologic studies show a largeabdominopelvic/retroperitoneal mass measuring 34.0 cm in greatest dimension andcompressing the inferior vena cava. The resected mass weighs 17 kilograms and displays alobulated yellow cut surface.

Master List 1700 Myxoid liposarcoma1511 Pleomorphic liposarcoma5606 Renal angiomyolipoma2256 Well-differentiated liposarcoma

Criteria for

Diagnosis

and

Comments

Histologic sections show relatively mature adipocytes of varying size, some of which contain atypical and hyperchromatic nuclei. Interspersed fibrotic bands containing scattered spindle and multipolar stromal cells with hyperchromatic nuclei are present. Depending on the area sampled, varying numbers of univacuolated and multivacuolated lipoblasts are identified. These findings are consistent with well-differentiated liposarcoma (WD-LPS).

WD-LPS is a locally aggressive neoplasm that presents as a deep-seated painless enlarging mass that can grow slowly to attain a very large size. These tumors occur in middle-aged adults with peak incidence in the 6th decade, with equal distribution in males and females. The most common sites of involvement are the deep soft tissues of the limbs, especially the thigh, followed by the retroperitoneum, the paratesticular area, and the mediastinum. Retroperitoneal tumors are often asymptomatic until the tumor has exceeded 20.0 cm in diameter. The term "well-differentiated liposarcoma" is used almost exclusively for tumors arising in the retroperitoneum, mediastinum, and pelvis, while the term "atypical lipomatous tumor" is used for those arising in the extremities. They are identical morphologically, karyotypically, and in terms of biologic behavior. However, retroperitoneal tumors are associated with a higher rate of recurrences largely due to difficulty in obtaining clear margins surgically.

On gross examination, WD-LPS is typically a large, well-circumscribed lobulated mass, with yellow to white firm cut surface depending on the proportion of mature adipocytic, fibrotic, and myxoid areas. Areas of fat necrosis are typical of large tumors. Histologically, WD-LPSs are composed predominantly of mature adipocytes showing significant variation in cell size and at

70

Page 71: 2019 JPIP-A 病理診断生涯教育プログラムLeslie Dodd, MD FCAP Sabrina C. Sopha, MD FCAP Raul Gonzalez, MD FCAP Nichole L. Steidler, MD FCAP Christina Isacson, MD FCAP Justin

# 2019-08

Well-differentiated liposarcoma

7060

34.0cm17kg

(1700) Myxoid liposarcoma(1511) Pleomorphic liposarcoma(5606) Renal angiomyolipoma(2256) Well-differentiated liposarcoma

lipoblast well-differentiated liposarcoma WD-LPS

50

20 cm

atypical lipomatous tumor

71

Page 72: 2019 JPIP-A 病理診断生涯教育プログラムLeslie Dodd, MD FCAP Sabrina C. Sopha, MD FCAP Raul Gonzalez, MD FCAP Nichole L. Steidler, MD FCAP Christina Isacson, MD FCAP Justin

least focal nuclear atypia. Scattered hyperchromatic and multinucleated stromal cells and varying numbers of lipoblasts are often present. Historically, WD-LPSs have been divided into lipoma-like, sclerosing, and inflammatory subtypes but the presence of more than one morphologic pattern is common, particularly in the retroperitoneal tumors. The characteristic MDM2 and CDK4 gene amplification on chromosome 12 seen in nearly all WD-LPSs can be highlighted with FISH studies. Immunohistochemistry can also be used as an adjunct tool with overexpression of the corresponding proteins manifesting as nuclear positivity.

Surgical resection with clear and wide margins is the treatment of choice for WD-LPS. The most important prognostic factor is the anatomic location; tumors occurring in surgically amenable sites such as soft tissue of the extremities have low rates of recurrence following complete excision with clear margins while those occurring in deep anatomic sites such as retroperitoneum tend to recur repeatedly and may result in death due to local effects or dedifferentiation and potential metastases. The term “dedifferentiation” is applied when a lipogenic tumor shows progression either in the primary or subsequent recurrence to a higher grade sarcoma with no obvious morphologic line of differentiation. The risk of dedifferentiation varies with size, location, and duration of the pre-existing tumor, but overall the estimated frequency of dedifferentiation is 10% with > 20% reported for retroperitoneal tumors.

The overall mortality for WD-LPS ranges from 0% for atypical lipomatous tumor of the extremities to > 80% for those of retroperitoneum in patients followed for 10 - 20 years.

Myxoid liposarcoma (MLS) makes up approximately 5% - 10% of sarcomas and tends to occur in younger patients. MLS is most commonly located in the deep soft tissues of the extremities and is very rarely seen in the retroperitoneum. Most MLS are relatively paucicellular with abundant myxoid background with a rich capillary network, often assuming a “pulmonary edema” pattern. The low-power vascular pattern sets MLS apart from other myxoid tumors. While the cellularity is relatively low, the cells that are present tend to be small, round with minimal mitotic activity. In addition, there are usually numerous lipoblasts. The presence of a prominent vascular pattern is helpful in differentiating MLS from dedifferentiated LPS with myxoid change. In addition, the majority of MLSs are characterized by a FUS-DDIT3 gene fusion, a feature that can be exploited for diagnostically difficult cases.

Pleomorphic liposarcoma (PLS) is the rarest subtype accounting for about 5% of all liposarcomas and occurs most commonly in the extremities while the trunk, retroperitoneum and spermatic cord are less frequently affected. Histologically, PLS has infiltrative margins and contain a varying number of lipoblasts in a background of a highgrade pleomorphic sarcoma. The presence of lipoblasts is essential for diagnosis highlighting the importance of adequate sampling. In most cases, the non-lipogenic component resembles undifferentiated pleomorphic sarcoma with spindle and multinucleated giant cells some of which are extremely large and often contain clear or vacuolated cytoplasm. The cytogenetic profile of PLS resembles those of other pleomorphic sarcomas rather than that of WD-LPS/dedifferentiated LPS.

72

Page 73: 2019 JPIP-A 病理診断生涯教育プログラムLeslie Dodd, MD FCAP Sabrina C. Sopha, MD FCAP Raul Gonzalez, MD FCAP Nichole L. Steidler, MD FCAP Christina Isacson, MD FCAP Justin

lipoma-like sclerosinginflammatory

12 MDM2CDK4 FISH

dedifferentiation

10% 20%

10 200 80

mxoid liposarcoma MLS 5 10

FUS-DDIT3

pleomorphic liposarcoma PLS 5%

73

Page 74: 2019 JPIP-A 病理診断生涯教育プログラムLeslie Dodd, MD FCAP Sabrina C. Sopha, MD FCAP Raul Gonzalez, MD FCAP Nichole L. Steidler, MD FCAP Christina Isacson, MD FCAP Justin

Renal angiomyolipoma (AML) belongs to the family of perivascular epithelioid cell tumors (PEComas) and presents as a yellow to gray mass varying in size from a few centimeters to 20.0 cm or more. The mass is usually well-delineated from the adjacent renal parenchyma. Microscopically, AML is composed of 3 tissue elements varying greatly: mature adipose tissue, thick-walled convoluted blood vessels, and sheets and interlacing bundles of smooth muscle often with a perivascular arrangement. The smooth muscle of AML expresses HMB45 in addition to actin and desmin. Tumors with a prominent fatty component may be mistaken for liposarcoma; however, the presence of thick-walled convoluted blood vessels and reactivity for HMB45 allow for their distinction.

Educational

Questions

1. Which of the following is most distinctive of well-differentiated liposarcoma (WD-LPS)?3343 FUS-DDIT3 gene fusion6080 Immunoreactivity for HMB456104 MDM2 gene amplification6105 Thick-walled convoluted blood vessels5942 Translocations involving chromosome 12

2. Which of the following is true of WD-LPS?6106 Dedifferentiation results in a distinct change in the cytogenetic profile.6107 Distant metastases are common.6108 Prognosis is independent of anatomic location.6109 Surgical resection with clear margins is the treatment of choice.6110 Tumors occurring in the extremities have higher rate of recurrence.

3. Which of the following is most typical of pleomorphic liposarcoma?6111 Areas of WD-LPS at the periphery6112 CDK4 gene amplification on chromosome 125945 Hypocellularity with prominent myxoid background with a rich capillary network6113 Non-lipogenic component resembling undifferentiated pleomorphic sarcoma6114 Well-circumscribed margins histologically

74

Page 75: 2019 JPIP-A 病理診断生涯教育プログラムLeslie Dodd, MD FCAP Sabrina C. Sopha, MD FCAP Raul Gonzalez, MD FCAP Nichole L. Steidler, MD FCAP Christina Isacson, MD FCAP Justin

renal angiomyolipoma AML perivascular epithelioid cell tumor: PEComa cm

20 cmAML 3

AMLHMB45

HMB45

1 31.

(3343) FUS-DDIT3(6080) HMB45(6104) MDM2(6105)(5942) 12

2. (6106)(6107)(6108)(6109)(6110)

3. (6111)(6112) 12 CDK4(5945)(6113)(6114)

97

75

Page 76: 2019 JPIP-A 病理診断生涯教育プログラムLeslie Dodd, MD FCAP Sabrina C. Sopha, MD FCAP Raul Gonzalez, MD FCAP Nichole L. Steidler, MD FCAP Christina Isacson, MD FCAP Justin

References 1. Crago A, Singer S. Clinical and molecular approaches to well-differentiated and dedifferentiated liposarcoma. Curr Opin Oncol. 2011; 23(4):373–378.

2. Dei Tos AP, PedutourRosen F. Atypical lipomatous tumour. In: Fletcher CDM, Bridge JA, Hagendoom PCW, Mertens F, eds. WHO Classification of Tumours of Soft Tissue and Bone. 4th ed. Lyon, FR: IARC; 2013:33-43.

3. Matthyssens LE, Creytens D, Ceelen WP. Retroperitoneal liposarcoma: current insights in diagnosis and treatment. Front Surg. 2015;2:4. Published online 2015 Feb 10. doi: 10.3389/fsurg.2015.00004.

4. Mavrogenis AF, Papagelopoulos PJ. Well-differentiated liposarcoma. Atlas Genet Cytogenet Oncol Haematol. 2013;17(8):579-582.

5. Vijay A, Ram L Retroperitoneal liposarcoma: a comprehensive review. Am J Clin Oncol. 2015;38(2):213-219.

6. Ware PL, Snow AN, Gvalani M, Pettenati MJ, Qasem SA. MDM2 copy numbers in well-differentiated and dedifferentiated liposarcoma: characterizing progression to high-grade tumors. Am J Clin Pathol. 2014;141(3):334-341.

Vijaya B. Reddy, MD

Surgical Pathology Committee

Rush University Medical Center

Chicago, IL

76

Page 77: 2019 JPIP-A 病理診断生涯教育プログラムLeslie Dodd, MD FCAP Sabrina C. Sopha, MD FCAP Raul Gonzalez, MD FCAP Nichole L. Steidler, MD FCAP Christina Isacson, MD FCAP Justin

1. Crago A, Singer S. Clinical and molecular approaches to well-differentiated and dedifferentiated liposarcoma. Curr Opin Oncol. 2011; 23(4):373–378.

2. Dei Tos AP, PedutourRosen F. Atypical lipomatous tumour. In: Fletcher CDM, Bridge JA, Hagendoom PCW, Mertens F, eds. WHO Classification of Tumours of Soft Tissue and Bone. 4th ed. Lyon, FR: IARC; 2013:33-43.

3. Matthyssens LE, Creytens D, Ceelen WP. Retroperitoneal liposarcoma: current insights in diagnosis and treatment. Front Surg. 2015;2:4. Published online 2015 Feb 10. doi: 10.3389/fsurg.2015.00004.

4. Mavrogenis AF, Papagelopoulos PJ. Well-differentiated liposarcoma. Atlas Genet Cytogenet Oncol Haematol. 2013;17(8):579-582.

5. Vijay A, Ram L Retroperitoneal liposarcoma: a comprehensive review. Am J Clin Oncol. 2015;38(2):213-219.

6. Ware PL, Snow AN, Gvalani M, Pettenati MJ, Qasem SA. MDM2 copy numbers in well-differentiated and dedifferentiated liposarcoma: characterizing progression to high-grade tumors. Am J Clin Pathol. 2014;141(3):334-341.

Vijaya B. Reddy, MD

Surgical Pathology Committee

Rush University Medical Center

Chicago, IL

77

Page 78: 2019 JPIP-A 病理診断生涯教育プログラムLeslie Dodd, MD FCAP Sabrina C. Sopha, MD FCAP Raul Gonzalez, MD FCAP Nichole L. Steidler, MD FCAP Christina Isacson, MD FCAP Justin

Case # 2019-09

Diagnosis Embryonal rhabdomyosarcoma

Site Uterine cervix

Clinical

Summary

The patient is a 17-year-old girl who presents with complaints of irregular vaginal bleeding. On clinical exam, she has a gelatinous multi-lobulated lesion protruding from the cervical os. The lesion is removed and submitted as a “cervical polyp” for histopathologic examination. The submitted tissue has a mucoid appearance and measures 19.0 cm in aggregate. Individual tissue fragments have a grape-like appearance.

Master List 2424 Adenosarcoma1698 Aggressive angiomyxoma1843 Alveolar rhabdomyosarcoma1951 Carcinosarcoma2468 Embryonal rhabdomyosarcoma

Criteria for

Diagnosis

and

Comments

Sections show multiple polypoid fragments of edematous, hemorrhagic, and focally necrotic soft tissue with occasional entrapped endocervical glands. The glandular epithelium is bland endocervix with some areas of squamous metaplasia. The stromal component shows some hemorrhage and normal-appearing small vessels. Scattered throughout the edematous stroma are small round to spindled cells with indistinct cytoplasm. Near the epithelial surfaces there is dense condensation of the small cell component into a distinctive cambium layer.

Embryonal rhabdomyosarcoma (ERMS) represents the most common subtype of rhabdomyosarcoma and the most frequent sarcoma of the pediatric population. The majority of ERMS occur in sites other than soft tissue; the head and neck region and the genitourinary or gynecologic tracts are particularly favored areas. In the latter site, there is a special variant of ERMS known as sarcoma botryoides or botryoid rhabdomyosarcoma that typically presents as a fleshy, "bag of grapes"-like lesion on clinical exam.

Sarcoma botryoides represents the most common malignant mesenchymal lesion of the cervix and is most often identified in young women between the ages of 12 – 26 years (average age: 18 years), with rare cases in older women. It frequently appears as a polyp-like lesion that protrudes through the cervical os or occasionally into the introitus. The prognosis for this variant of rhabdomyosarcoma is generally favorable. Patients of child-bearing age can often be treated with a fertility-sparing procedure.

Immunohistochemically, ERMS will express markers of muscle differentiation including desmin, actins, myogenin, and MyoD1. ERMS as a group do not show specific reproducible cytogenetic or molecular aberrations that translate into an ancillary diagnostic test. Structural

78

Page 79: 2019 JPIP-A 病理診断生涯教育プログラムLeslie Dodd, MD FCAP Sabrina C. Sopha, MD FCAP Raul Gonzalez, MD FCAP Nichole L. Steidler, MD FCAP Christina Isacson, MD FCAP Justin

# 2019-09

Embryonal rhabdomyosarcoma

17

19 cm

(2424) Adenosarcoma(1698) Aggressive angiomyxoma(1843) Alveolar rhabdomyosarcoma(1951) Carcinosarcoma(2468) Embryonal rhabdomyosarcoma

cambium layer

embryonal rhabdomyosarcoma ERMSERMS

ERMSsarcoma botryoides botryoid

rhabdomyosarcoma

sarcoma botryoides 1226 18

ERMSMyoD1 ERMS

2 8 13

79

Page 80: 2019 JPIP-A 病理診断生涯教育プログラムLeslie Dodd, MD FCAP Sabrina C. Sopha, MD FCAP Raul Gonzalez, MD FCAP Nichole L. Steidler, MD FCAP Christina Isacson, MD FCAP Justin

and numeric copy number changes of chromosomes 2, 8, and 13 are frequent findings.

The differential diagnosis of ERMS/sarcoma botryoides includes other stromal and mixed epithelial and stromal neoplasms of the gynecologic tract.

Aggressive angiomyxoma (also called deep angiomyxoma) is a neoplasm that tends to arise in the deeper soft tissues of the pelvic or perineal regions of adult women. Aggressive/deep angiomyxomas are lobulated lesions that tend to grow around other structures (rectum, vagina) but not infiltrate. They are hypocellular and contain an abundance of matrix that appears myxoid but actually represents intralesional edema. Scattered throughout the lesion are a few small to medium sized vessels with very thin walls. The predominant cell type is a very small spindle cell with indistinct cytoplasmic processes. Nuclear atypia and pleomorphism are absent.

Unlike ERMS, aggressive/deep angiomyxomas are frequently immunoreactive for estrogen receptors (ER) and progesterone receptors (PR). They may also express smooth muscle actin but desmin staining is usually negative. Although they do not metastasize, they have a propensity to recur and may be associated with morbidity because of their deep-seated location. Because these tumors express ER and PR, patients can be treated with hormone agonists and can often delay surgical excision.

Adenosarcomas may closely mimic sarcoma botryoides, but they tend to occur in older women (average age: 58 years). They most frequently arise in the endometrium, with rare cases arising in the cervix. They commonly present with abnormal bleeding and generally have a polypoid appearance. Unlike sarcoma botryoides, they lack the “bag of grapes” gross appearance. The overall prognosis for adenosarcoma is fair. Patients with tumor confined to the endometrium tend to do well after hysterectomy. Women with higher-stage lesions are more likely to experience recurrence and metastases.

On histologic examination, adenosarcomas tend to have a phyllodes or “leaf-like” pattern. They are biphasic neoplasms comprised of a benign epithelial component (usually endometrial-type glands) and a spindled stromal component. The stroma may condense around the epithelial glands, creating a “cambium layer-like” configuration. The stroma of adenosarcomas is usually bland but can occasionally become pleomorphic and, in some instances contain rhabdomyoblasts. The latter will express muscle markers including desmin and myogenin. Differentiating adenosarcoma from ERMS may be difficult because of these histologic similarities. In general, ERMS arise in young patients whereas adenosarcomas typically occur in older women. In addition, adenosarcomas usually originate in the endometrium, where ERMS seldom occurs. Benign glands are an integral feature of adenosarcoma but are often present as entrapped elements in rhabdomyosarcoma. In addition, the mesenchymal element of adenosarcoma will express CD10, WT1, ER, and PR, whereas ERMS seldom do.

80

Page 81: 2019 JPIP-A 病理診断生涯教育プログラムLeslie Dodd, MD FCAP Sabrina C. Sopha, MD FCAP Raul Gonzalez, MD FCAP Nichole L. Steidler, MD FCAP Christina Isacson, MD FCAP Justin

ERMS

aggressive angiomyxoma deep angiomyxoma

ERMSER PR

ER PR

adenosarcoma58

phyllodes leaf-like

cambium layer-like configuration

ERMSERMS

ERMS

CD10 WT1 ER PR ERMS

81

Page 82: 2019 JPIP-A 病理診断生涯教育プログラムLeslie Dodd, MD FCAP Sabrina C. Sopha, MD FCAP Raul Gonzalez, MD FCAP Nichole L. Steidler, MD FCAP Christina Isacson, MD FCAP Justin

Alveolar rhabdomyosarcoma (ARMS) is the second most common subtype of rhabdomyosarcoma, accounting for about 20% of all cases. This subtype is usually seen in a slightly older population than the embryonal subtype, but there is a significant age overlap of patients. Although ARMS is more likely to arise in the soft tissue of the extremities or paraspinal region, it may occasionally arise in the pelvis.

In its classic form, ARMS is characterized by small nests of tumor cells separated by thin fibrovascular septae. The cells themselves are small, primitive-appearing rhabdomyoblasts. Pleomorphism is usually more apparent in alveolar rhabdomyosarcoma and large cells with abundant eosinophilic cytoplasm, occasionally with visible cross striations (rhabdomyoblasts), are often found. However, ARMS is histologically variable and can often closely resemble ERMS, especially with the so-called “solid” variant of ARMS. In this subtype, the characteristic fibrovascular septae are absent, resulting in a solid sheet of tumor cells with very little rhabdomyoblastic differentiation. Like ERMS, ARMS will stain with markers of muscle differentiation.

ARMS is an aggressive neoplasm treated with a multimodal approach. Individuals with ARMS tend to be higher stage at presentation and the tumor is often metastatic by the time it is discovered. Because the treatment for ERMS and ARMS differ significantly, it is important to establish a definitive diagnosis of ARMS prior to initiation of therapeutic intervention. Fortunately, ARMS is characterized by a recurrent translocation in about 80% of cases. This involves a fusion of the FOXO1 gene (formerly known as FKHR for forkhead in rhabdomyosarcoma) with either a PAX3 (t(2;13), most common) or PAX7 (t(1;13)) gene. Because of the differences in prognosis, current recommendations include molecularconfirmation of rhabdomyosarcoma subtype whenever possible.

Carcinosarcoma, also known as malignant mixed Müllerian tumor (MMMT), is another malignancy of the gynecologic tract that may resemble sarcoma botryoides. Carcinosarcoma occurs almost exclusively in postmenopausal women and preferentially arises in the endometrium. Patients present with abnormal bleeding, an enlarged uterus and elevated CA125. On histology, carcinosarcoma is characterized by the presence of both a malignant epithelial component (carcinoma) as well as a malignant sarcomatous stroma. The carcinomatous element is usually endometrioid or serous but occasionally can be squamous, clear cell, or another subtype. The sarcomatous component is most often high grade. About 50% of carcinosarcomas will show heterologous differentiation which manifests as foci of rhabdomyoblastic (most common), chondroblastic, osteosarcomatous, or liposarcomatousdifferentiation. The rhabdomyoblastic elements will stain for muscle markers, but theremainder of the stroma will typically express CD10, p16, and WT1, markers that are usually negative in ERMS. The prognosis for carcinosarcoma is poor and it is treated aggressively with surgery and adjuvant therapy. Survival rates are related to stage at presentation.

82

Page 83: 2019 JPIP-A 病理診断生涯教育プログラムLeslie Dodd, MD FCAP Sabrina C. Sopha, MD FCAP Raul Gonzalez, MD FCAP Nichole L. Steidler, MD FCAP Christina Isacson, MD FCAP Justin

alveolar rhabdomyosarcoma ARMS20

ARMS

ARMSARMS

ARMS ARMS solid variantERMS ARMS

ERMSARMS

ARMS ARMSERMS ARMS

ARMSARMS 80

FOXO1 FKHR forkhead in rhabdomyosarcomaPAX3 t (2;13) PAX7 t (1;13)

carcinosarcoma malignant mixed Müllerian tumor MMMT

CA125

50

CD10 p16 WT1 ERMS

83

Page 84: 2019 JPIP-A 病理診断生涯教育プログラムLeslie Dodd, MD FCAP Sabrina C. Sopha, MD FCAP Raul Gonzalez, MD FCAP Nichole L. Steidler, MD FCAP Christina Isacson, MD FCAP Justin

Educational

Questions

1. Which of the following features best distinguishes embryonal rhabdomyosarcoma (botryoid subtype) from adenosarcoma?

6115 Absence of staining for CD10, WT1, ER, and PR6116 Positive staining for desmin and myogenin6117 Presence of a condensed layer of cells around glands6118 Presence of a reproducible cytogenetic translocation6119 Presence of rhabdomyoblasts

2. Which of the following features best separates embryonal subtype of rhabdomyosarcoma (ERMS) from the alveolar subtype of rhabdomyosarcoma (ARMS)?

6120 ARMS has an alveolar pattern with fibrovascular septae.6121 ARMS has a characteristic gene translocation.6122 ARMS is a soft tissue-based lesion.6123 ERMS is more likely to have numerous rhabdomyoblasts.6124 ERMS is more pleomorphic than ARMS.

3. Which of the following scenarios is most consistent with sarcoma botryoides?6125 A 15-year-old with a pelvic mass and metastases to the lung6126 A 25-year-old with a deep soft tissue lesion surrounding the rectum6127 A 25-year-old with a polypoid lesion protruding through the cervical os6128 A 35-year-old with an endometrial polyp and abnormal bleeding6129 A 65-year-old with uterine enlargement and abnormal bleeding

84

Page 85: 2019 JPIP-A 病理診断生涯教育プログラムLeslie Dodd, MD FCAP Sabrina C. Sopha, MD FCAP Raul Gonzalez, MD FCAP Nichole L. Steidler, MD FCAP Christina Isacson, MD FCAP Justin

1 31.

(6115) CD10 WT1 ER PR(6116)(6117)(6118)(6119)

2. ERMS ARMS(6120) ARMS(6121) ARMS(6122) ARMS(6123) ERMS(6124) ERMS ARMS

3.(6125) 15(6126) 25(6127) 25(6128) 35(6129) 65

97

85

Page 86: 2019 JPIP-A 病理診断生涯教育プログラムLeslie Dodd, MD FCAP Sabrina C. Sopha, MD FCAP Raul Gonzalez, MD FCAP Nichole L. Steidler, MD FCAP Christina Isacson, MD FCAP Justin

References 1. Dehner LP, Jarzembowski JA, Hill DA. Embryonal rhabdomyosarcoma of the uterine cervix: a report of 14 cases and a discussion of its unusual clinicopathological associations. Mod Pathol. 2012;25(4):602-614.2. Li RF, Gupta M, McCluggage WG, Ronnett BM. Embryonal rhabdomyosarcoma (botryoid type) of the uterine corpus and cervix in adult women: report of a case series and review of the literature. Am J Surg Pathol. 2013;37(3):344-355.3. McCluggage WG. Mullerian adenosarcoma of the female genital tract. Adv Anat Pathol. 2010;17(2):122-129.4. Parham DM, Barr FG. Classification of rhabdomyosarcoma and its molecular basis. Adv Anat Pathol. 2013;20(6):387-397.5. Rudzinski ER, Teot LA, Anderson JR, et al. Dense pattern of embryonal rhabdomyosarcoma, a lesion easily confused with alveolar rhabdomyosarcoma: a report from the Soft Tissue Sarcoma Committee of the Children’s Oncology Group. Am J Clin Pathol. 2013;140(1):82-90.6. Sharma NK, Sorosky JI, Bender D, Fletcher MS, Sood AK. Malignant mixed mullerian tumors (MMMT) of the cervix. Gynecol Oncol. 2005;97:442-445.7. Soslow RA, Ali A, Oliva E. Mullerian adenosarcomas: an immunophenotypic analysis of 35 cases. Am J Surg Pathol. 2008;32(7):1013-1021.8. van Roggen JF, van Unnik JA, Briaire-de Bruijn IH, Hogendoorn PC. Aggressive angiomyxoma: a clinicopathological and immunohistochemical study of 11 cases with long-term follow-up. Virchows Arch. 2005;446(2):157-163.9. Wexler LH, Ladanyi M. Diagnosing alveolar rhabdomyosarcoma: morphology must be coupled with fusion confirmation. J Clin Oncol. 2010;28(13):2126-2128.

Leslie G Dodd, MDSurgical Pathology Committee

University of North CarolinaChapel Hill, NC

86

Page 87: 2019 JPIP-A 病理診断生涯教育プログラムLeslie Dodd, MD FCAP Sabrina C. Sopha, MD FCAP Raul Gonzalez, MD FCAP Nichole L. Steidler, MD FCAP Christina Isacson, MD FCAP Justin

1. Dehner LP, Jarzembowski JA, Hill DA. Embryonal rhabdomyosarcoma of the uterine cervix: a report of 14 cases and a discussion of its unusual clinicopathological associations. Mod Pathol.2012;25(4):602-614.2. Li RF, Gupta M, McCluggage WG, Ronnett BM. Embryonal rhabdomyosarcoma (botryoid type) of the uterine corpus and cervix in adult women: report of a case series and review of the literature. Am J Surg Pathol. 2013;37(3):344-355.3. McCluggage WG. Mullerian adenosarcoma of the female genital tract. Adv Anat Pathol. 2010;17(2):122-129.4. Parham DM, Barr FG. Classification of rhabdomyosarcoma and its molecular basis. Adv Anat Pathol. 2013;20(6):387-397.5. Rudzinski ER, Teot LA, Anderson JR, et al. Dense pattern of embryonal rhabdomyosarcoma, a lesion easily confused with alveolar rhabdomyosarcoma: a report from the Soft Tissue Sarcoma Committee of the Children’s Oncology Group. Am J Clin Pathol. 2013;140(1):82-90.6. Sharma NK, Sorosky JI, Bender D, Fletcher MS, Sood AK. Malignant mixed mullerian tumors (MMMT) of the cervix. Gynecol Oncol. 2005;97:442-445.7. Soslow RA, Ali A, Oliva E. Mullerian adenosarcomas: an immunophenotypic analysis of 35 cases. Am J Surg Pathol. 2008;32(7):1013-1021.8. van Roggen JF, van Unnik JA, Briaire-de Bruijn IH, Hogendoorn PC. Aggressive angiomyxoma: a clinicopathological and immunohistochemical study of 11 cases with long-term follow-up. Virchows Arch. 2005;446(2):157-163.9. Wexler LH, Ladanyi M. Diagnosing alveolar rhabdomyosarcoma: morphology must be coupled with fusion confirmation. J Clin Oncol. 2010;28(13):2126-2128.

Leslie G Dodd, MDSurgical Pathology Committee

University of North CarolinaChapel Hill, NC

87

Page 88: 2019 JPIP-A 病理診断生涯教育プログラムLeslie Dodd, MD FCAP Sabrina C. Sopha, MD FCAP Raul Gonzalez, MD FCAP Nichole L. Steidler, MD FCAP Christina Isacson, MD FCAP Justin

Case # 2019-10

Diagnosis Anaplastic thyroid carcinoma

Site Anterior neck

Clinical

Summary

A 68-year-old woman presents with a rapidly enlarging neck mass of one month’s duration. She also has recent onset dysphagia, dysphonia, and significant weight loss. Clinical examination reveals a firm, fixed mass in the anterior neck, and imaging reveals a large mass centered on the thyroid gland with associated cervical lymphadenopathy. The patient undergoes a surgical resection of the neck mass. Gross examination reveals a firm nodular mass replacing the thyroid and measuring 9.5 x 8.3 x 5.4 cm. Areas of necrosis are present.

Master List 5609 Anaplastic thyroid carcinoma1620 Malignant melanoma2048 Medullary thyroid carcinoma5610 Pleomorphic undifferentiated sarcoma2272 Poorly differentiated thyroid carcinoma1600 Riedel thyroiditis

Criteria for

Diagnosis

and

Comments

Microscopic sections show a highly pleomorphic neoplasm with extensive necrosis.The tumor cells are arranged in sheets and have epithelioid and spindle cell morphology. There are also numerous pleomorphic tumor giant cells. Some sections show vascular invasion. Depending on the areas sampled, cells with abundant pink cytoplasm and eccentrically placed nuclei suggestive of rhabdoid morphology can be seen. In some sections, differentiated thyroid carcinoma with oncocytic features can be seen at the periphery of the anaplastic tumor. The histological features together with the clinical presentation are consistent with the diagnosis of an anaplastic thyroid carcinoma.

Anaplastic thyroid carcinoma is a highly aggressive thyroid tumor with a median 1-year survival rate of only 10%-20%. The tumor usually presents as a rapidly enlarging neck mass in older individuals. Patients may have a history of prior welldifferentiated thyroid carcinoma, which is usually papillary carcinoma but also may be follicular or Hürthle cell carcinoma. Patients may present with symptoms such as difficulty breathing, dysphagia, and hoarseness because of the infiltration of neck structures by a rapidly growing tumor. Not uncommonly, patients present in an advanced stage with nodal or even distant metastasis.

Macroscopically, anaplastic carcinomas are light tan tumors which replace the thyroid parenchyma. The tumor is generally nodular, ill-defined, and fibrotic. Areas of hemorrhage and necrosis can often be identified grossly. Extrathyroidal extension is a common finding. There may be infiltration of adjacent structures.

88

Page 89: 2019 JPIP-A 病理診断生涯教育プログラムLeslie Dodd, MD FCAP Sabrina C. Sopha, MD FCAP Raul Gonzalez, MD FCAP Nichole L. Steidler, MD FCAP Christina Isacson, MD FCAP Justin

# 2019-10

Anaplastic thyroid carcinoma

68 1

9.5 x 8.3 x 5.4cm

(5609) Anaplastic thyroid carcinoma(1620) Malignant melanoma(2048) Medullary thyroid carcinoma(5610) Pleomorphic undifferentiated sarcoma(2272) Poorly differentiated thyroid carcinoma(1600) Riedel thyroiditis

anaplastic thyroid carcinoma 110 20

Hürthle

89

Page 90: 2019 JPIP-A 病理診断生涯教育プログラムLeslie Dodd, MD FCAP Sabrina C. Sopha, MD FCAP Raul Gonzalez, MD FCAP Nichole L. Steidler, MD FCAP Christina Isacson, MD FCAP Justin

Microscopically, anaplastic carcinoma can have varied appearances. The most common patterns include sarcomatoid, giant cell, or epithelioid patterns. Irrespective of the pattern, they are characterized by cells with marked nuclear pleomorphism, high mitotic activity, and geographic areas of necrosis. They have a highly invasive pattern of growth. Less common patterns include osteoclastic giant cell-rich type, angiomatoid, rhabdoid, lymphoepithelioma-like, and paucicellular type. Mixed patterns can also be seen. Heterologous differentiation including bone or cartilage can be seen. Extensive sampling may be necessary to demonstrate areas of differentiated carcinoma which, when present, may help distinguish anaplastic thyroid carcinoma from a primary sarcoma of the thyroid or metastatic sarcomas.

The sarcomatoid pattern mimics a high-grade pleomorphic sarcoma composed of atypical spindle cells arranged in sheets or fascicles. The giant cell pattern shows large pleomorphic tumor cells, some of which can be multinucleated. An epithelioid pattern resembles epithelial cells and can show squamous differentiation.

Although the cell of origin for anaplastic thyroid carcinoma is the thyroid follicular epithelial cell, the tumor cells often lose reactivity for thyroid-specific immune markers such as TTF-1and thyroglobulin. They may retain positivity for PAX8, though this reactivity may be focal. These tumors generally retain cytokeratin positivity, which helps to establish the epithelial nature of the tumor.

Anaplastic thyroid carcinoma is most consistently associated with TP53 gene mutations in around 40%-80% of patients. Mutations in BRAF and RAS genes have also been identified, similar to those present in well-differentiated tumors. Additional genetic alterations including mutations of beta-catenin (CTNNB1), PTEN, PI3KCA, and TERT genes have been identified and associated with the aggressive nature of the tumor.

Malignant melanoma, a mimicker of many high-grade malignancies, can have similarmorphology to anaplastic thyroid carcinoma. Spindled and epithelioid morphology can be seen. However, immunohistochemical markers for melanoma including S100, HMB45, and Melan-A (positive in melanoma, negative in anaplastic thyroid carcinoma) are useful adjuncts to clinical history in establishing the diagnosis.

Medullary thyroid carcinoma, particularly with prominent spindle cell morphology, can bedifficult to differentiate from anaplastic thyroid carcinoma based on histological appearance alone. Personal or family history of multiple endocrine neoplasia, a younger age at presentation, or the presence of amyloid on histological sections prompts a strong suspicion for medullary thyroid carcinoma. Immunohistochemical studies are confirmatory. A Congo Red special stain may show the characteristic “apple-green” birefringence on polarization of areas containing amyloid. Medullary thyroid carcinomas are positive for calcitonin, synaptophysin, chromogranin, and TTF-1.

Pleomorphic undifferentiated sarcoma, although very rare, can occur in the thyroid gland and

90

Page 91: 2019 JPIP-A 病理診断生涯教育プログラムLeslie Dodd, MD FCAP Sabrina C. Sopha, MD FCAP Raul Gonzalez, MD FCAP Nichole L. Steidler, MD FCAP Christina Isacson, MD FCAP Justin

sarcomatoidgiant cell epithelioid

osteoclastic giant cell-rich type angiomatoid rhabdoid lymphoepithelioma-like

paucicellular type

TTF-1PAX8

TP53 40 80BRAF RAS

- CTNNB1 PTEN PI3KCA TERT

malignant melanomaS100

HMB45 Melan-A

medullary thyroid carcinoma

TTF-1

pleomorphic undifferentiated sarcoma

91

Page 92: 2019 JPIP-A 病理診断生涯教育プログラムLeslie Dodd, MD FCAP Sabrina C. Sopha, MD FCAP Raul Gonzalez, MD FCAP Nichole L. Steidler, MD FCAP Christina Isacson, MD FCAP Justin

can be challenging to differentiate from sarcomatoid anaplastic thyroid carcinoma. The absence of residual well-differentiated thyroid carcinoma and absence of cytokeratin expression support the diagnosis of a primary sarcoma over anaplastic thyroid carcinoma.

Poorly differentiated (PD) thyroid carcinomas, also referred to as insular carcinomas, are a subset of thyroid malignancies which lie between well-differentiated carcinoma and anaplastic carcinoma in clinical behavior. Identifying them as a separate group from undifferentiated carcinoma is important since there are significant differences in treatment approaches and prognosis. Histologically, they are composed of a comparatively monomorphic population of cells arranged in nests, trabeculae, or cords. The Turin criteria for diagnosis of PD carcinoma require (1) presence of a solid/trabecular/insular pattern of growth, (2) absence of the conventional nuclear features of papillary carcinoma, and (3) presence of at least one of the following features: convoluted nuclei; mitotic activity >3 mitoses per high power field; andtumor necrosis. Anaplasia such as that seen in anaplastic thyroid carcinomas is absent. In addition, PD carcinomas generally retain positivity for TTF-1 and thyroglobulin, unlike anaplastic carcinoma.

Riedel thyroiditis is a sclerotic inflammatory process wherein keloid-like fibrotic stroma replaces the thyroid parenchyma in a background of inflammatory infiltrate. Extrathyroidal extension can also be seen. Riedel thyroiditis can mimic the paucicellular variant of anaplastic carcinoma both clinically and histologically. However, the paucicellular variant of anaplastic carcinoma has highly atypical tumor cells which are not seen in Riedel thyroiditis.

Educational

Questions

1. Which of the following immunohistochemical stains is most commonly positive inanaplastic thyroid carcinoma?

6130 Calcitonin1541 Chromogranin5030 PAX85031 Thyroglobulin3435 TTF-1

2. Which of the following variants of anaplastic thyroid carcinoma can mimic Riedel thyroiditis?6131 Angiomatoid1349 Lymphoepithelioma-like6132 Osteoclast-like giant cell-rich6133 Paucicellular6134 Sarcomatoid

92

Page 93: 2019 JPIP-A 病理診断生涯教育プログラムLeslie Dodd, MD FCAP Sabrina C. Sopha, MD FCAP Raul Gonzalez, MD FCAP Nichole L. Steidler, MD FCAP Christina Isacson, MD FCAP Justin

poorly differentiated thyroid carcinoma insular carcinoma

The Turin criteria (1)

(2) (3) 1convoluted nuclei 3 / 10

TTF-1

Riedel thyroiditis

1 31.

(6130) (1541) (5030) PAX8(5031) (3435) TTF-1

2. (6131) (1349) (6132) (6133) (6134)

93

Page 94: 2019 JPIP-A 病理診断生涯教育プログラムLeslie Dodd, MD FCAP Sabrina C. Sopha, MD FCAP Raul Gonzalez, MD FCAP Nichole L. Steidler, MD FCAP Christina Isacson, MD FCAP Justin

3. Which of the following gene mutations is associated with anaplastic thyroid carcinoma?3385 BRCA13735 KIT6135 PALB26136 STK115018 TP53

References 1. Acquaviva G, Visani M, Repaci A, et al. Molecular pathology of thyroid tumours of follicular cells: a review of genetic alterations and their clinicopathological relevance. Histopathology.2018;72(1):6-31.

2. Bishop JA, Sharma R, Westra WH. PAX8 immunostaining of anaplastic thyroid carcinoma: a reliable means of discerning thyroid origin for undifferentiated tumors of the head and neck. Hum Pathol. 2011;42(12):1873-1877.

3. Miettinen M, Franssila KO. Variable expression of keratins and nearly uniform lack of thyroid transcription factor 1 in thyroid anaplastic carcinoma. Hum Pathol. 2000;31(9):1139-1145.

4. Nikiforov Y, Biddinger PW, Thompson LDR. Diagnostic Pathology and Molecular Genetics of the Thyroid. Philadelphia, PA: Wolters Kluwer Health/Lippincott Williams & Wilkins; 2012.

5. Ragazzi M, Ciarrocchi A, Sancisi V, Gandolfi G, Bisagni A, Piana S. Update on anaplastic thyroid carcinoma: morphological, molecular, and genetic features of the most aggressive thyroid cancer. Int J of Endocrinol. 2014;2014:790834.

6. Rosai J. et al. Ch. 2, Tumours of the Thyroid. In: Lloyd RV, Osamura RY, Klöppel G, Rosai J, eds. WHO classification of tumours of endocrine organs. Lyon, FR. International Agency for Research on Cancer; 2017:65-143.

7. Volante M, Collini P, Nikiforov YE, et al. Poorly differentiated thyroid carcinoma: the Turin proposal for the use of uniform diagnostic criteria and an algorithmic diagnostic approach. Am J Surg Pathol. 2007;31(8):1256-1264.

Rohini Kannuswamy, MDSurgical Pathology Fellow

Duke UniversityDurham, NC

Xiaoyin “Sara” Jiang, MDSurgical Pathology Committee

Duke UniversityDurham, NC

94

Page 95: 2019 JPIP-A 病理診断生涯教育プログラムLeslie Dodd, MD FCAP Sabrina C. Sopha, MD FCAP Raul Gonzalez, MD FCAP Nichole L. Steidler, MD FCAP Christina Isacson, MD FCAP Justin

3(3385) BRCA1(3735) KIT(6135) PALB2(6136) STK11(5018) TP53

97

1. Acquaviva G, Visani M, Repaci A, et al. Molecular pathology of thyroid tumours of follicular cells: a review of genetic alterations and their clinicopathological relevance. Histopathology.2018;72(1):6-31.

2. Bishop JA, Sharma R, Westra WH. PAX8 immunostaining of anaplastic thyroid carcinoma: a reliable means of discerning thyroid origin for undifferentiated tumors of the head and neck. Hum Pathol. 2011;42(12):1873-1877.

3. Miettinen M, Franssila KO. Variable expression of keratins and nearly uniform lack of thyroid transcription factor 1 in thyroid anaplastic carcinoma. Hum Pathol. 2000;31(9):1139-1145.

4. Nikiforov Y, Biddinger PW, Thompson LDR. Diagnostic Pathology and Molecular Genetics of the Thyroid. Philadelphia, PA: Wolters Kluwer Health/Lippincott Williams & Wilkins; 2012.

5. Ragazzi M, Ciarrocchi A, Sancisi V, Gandolfi G, Bisagni A, Piana S. Update on anaplastic thyroid carcinoma: morphological, molecular, and genetic features of the most aggressive thyroid cancer. Int J of Endocrinol. 2014;2014:790834.

6. Rosai J. et al. Ch. 2, Tumours of the Thyroid. In: Lloyd RV, Osamura RY, Klöppel G, Rosai J, eds. WHO classification of tumours of endocrine organs. Lyon, FR. International Agency for Research on Cancer; 2017:65-143.

7. Volante M, Collini P, Nikiforov YE, et al. Poorly differentiated thyroid carcinoma: the Turin proposal for the use of uniform diagnostic criteria and an algorithmic diagnostic approach. Am J Surg Pathol. 2007;31(8):1256-1264.

Rohini Kannuswamy, MDSurgical Pathology Fellow

Duke UniversityDurham, NC

Xiaoyin “Sara” Jiang, MDSurgical Pathology Committee

Duke UniversityDurham, NC

95

Page 96: 2019 JPIP-A 病理診断生涯教育プログラムLeslie Dodd, MD FCAP Sabrina C. Sopha, MD FCAP Raul Gonzalez, MD FCAP Nichole L. Steidler, MD FCAP Christina Isacson, MD FCAP Justin

PIP-A 2019

Performance Improvement Program Critique

Educational Questions Answer Key

2019-011. 6020 It is used to designate Wilms tumors when the heterologous epithelial and stromal differentiation

comprise >50% of the tumor volume.2. 6021 Diffuse anaplasia3. 6023 Loss of imprinting on 11p15

2019-021. 1653 Sex cord-stromal tumors2. 6030 Meigs syndrome and Gorlin syndrome (basal cell nevus syndrome)3. 1687 Reticulin

2019-031. 6037 Type IIb glycogen storage disease (Danon disease)2. 6041 Stains for cytochrome oxidase and mitochondrial antigens are useful in making the diagnosis.3. 6046 Non-Mendelian

2019-041. 6051 TFE3 RCCs may mimic clear cell RCCs.2. 6057 PRCC fusion tumors show higher rates of cathepsin K expression than ASPL fusion

tumors.3. 6061 TFE3 immunophenotype is variable and depends on the fusion partner.

2019-051. 6065 HCCs are more frequently encountered in males and in African Americans.2. 6069 Neoplastic cells with eosinophilic cytoplasm with frequent pale inclusions and cytoplasmic

globules grow between dense fibrous septae.3. 6074 Collections of hepatocytes with a decreased cell volume, minimal nuclear pleomorphism, an

increased nucleus to cytoplasm ratio, and increased nuclear density2019-06

1. 6079 Immunoreactivity for p63 and p402. 6084 Stage for stage, survival rate is significantly better than for adenocarcinoma.3. 6087 Common alterations include EGFR, KRAS, and BRAF

2019-071. 1966 Seminoma2. 6096 Glypican 3 (-), OCT 3/4 (+), CD117 (+), D2-40 (+), CD30 (-)3. 6101 Intertubular growth (> 3 high power fields) is an adverse prognostic factor.

2019-081. 6104 MDM2 gene amplification2. 6109 Surgical resection with clear margins is the treatment of choice.3. 6113 Non-lipogenic component resembling undifferentiated pleomorphic sarcoma

2019-091. 6115 Absence of staining for CD10, WT1, ER, and PR2. 6121 ARMS has a characteristic gene translocation.3. 6127 A 25-year-old with a polypoid lesion protruding through the cervical os

2019-101. 5030 PAX82. 6133 Paucicellular3. 5018 TP53

96

Page 97: 2019 JPIP-A 病理診断生涯教育プログラムLeslie Dodd, MD FCAP Sabrina C. Sopha, MD FCAP Raul Gonzalez, MD FCAP Nichole L. Steidler, MD FCAP Christina Isacson, MD FCAP Justin

2019-011. (6020) 50

2. (6021)3. (6023) 11p15

2019-021. (1653)2. (6030) Meigs Gorlin3. (1687) Reticulin

2019-031. (6037) IIb Danon2. (6041)3. (6046)

2019-041. (6051) TFE3 RCC RCC2. (6057) PRCC ASPL K3. (6061) TFE3

2019-051. (6065) HCC2. (6069)

3. (6074) N/C

2019-061. (6079) p63 p402. (6084)3. (6087) EGFR KRAS BRAF

2019-071. (1966) Seminoma2. (6096) Glypican 3 (-), OCT 3/4 (+), CD117 (+), D2-40 (+), CD30 (-)3. (6101) 4 HPF

2019-081. (6104) MDM22. (6109)3. (6113)

2019-091. (6115) CD10 WT1 ER PR2. (6121) ARMS3. (6127) 25

2019-101. (5030) PAX82. (6133)3. (5018) TP53

97

Page 98: 2019 JPIP-A 病理診断生涯教育プログラムLeslie Dodd, MD FCAP Sabrina C. Sopha, MD FCAP Raul Gonzalez, MD FCAP Nichole L. Steidler, MD FCAP Christina Isacson, MD FCAP Justin

2019 JPIP-A 病理診断生涯教育プログラム Performance Improvement Program in Surgical Pathology(PIP) 2019年6月 発行 編集・発行 株式会社CGI 〒106-0041 東京都港区麻布台 1-7-2 神谷町麻布台ビル ホームページ http://www.cgikk.com 電 話 03-5563-1368(代表) 本プログラムに関するお問い合わせ先 電話 03-5563-1327 E-mail [email protected]